Download as pdf or txt
Download as pdf or txt
You are on page 1of 103

SHANKAR TM

IAS ACADEMY
^ The Best IAS Academy In South India SINCE 2004

PRESTORMING 2020
TEST - 9
GEOGRAPHY II - EXPLANATIONS

INDEX
QUESTION NO PAGE NO

1. 4
2. 5
3. 5
4. 6
5. 8
6. 9
7. 10
8. 11
9. 11
10. 12
11. 14
12. 14
13. 15
14. 16
15. 17
16. 19
17. 22
18. 23
19 . 24
20. 24
21. 25
22. 26
23. 27
24. 28
25. 29
26. 29
27. 30
28. 31
29 . 32
30. 34

1
FREE BOOKS, NOTES & VIDEOS FOR CIVILSERVICES

EBOOKS & UPSC PRELIMS USPC MAINS VIDEO FOR DAILY


MAGZINES MATERIALS MATERIALS CIVILSERVICES NEWSAPERS

SECUREIAS UPSC PRELIMS UPSC MAINS DELHI CIVILSERVICES


TESTSERIES TESTSERIES STUDENTS BOOKS

OPTIONAL SUBJECTS BOOKS, STATE PCS, SSC, BANKING


TEST SERIES, VIDEOS & NOTES BOOKS, TESTS VIDEOS & NOTES
1.GEOGRAPHY 1.UPPSC 2.SSC 3.MPSC
2.HISTORY 4.IBPS 5.RAS & RPSC
3.MATHEMATICS ENGINEERING BOOKS & MATERIAL
4. SOCIOLOGY 1. IES 2. GATE 3. IFoS
5.PUBLIC ADMINISTRATION 4. COMPUTER SCIENCE
6. POLITICAL SCIENCE 5. MECHINICAL ENGINEERING
7. ECONOMICS OTHER TELEGRAM CHANNELS
8 PHYSICS 1 GOVERNMENT JOBS
9 COMMERCE ACCOUNTANCY 2 LEARN YOGA & MEDITATION
10 ANTHROPOLOGY 3 LEARN ENGLISH
11 LAW 4 BEST DELAS & OFFERS
12 PHILOSOPHY 5 IAS HINDI BOOKS
13 CHARTERED ACCOUNTANTANCY 6 PDFs FOR ALL EXAMS
14 MEDICAL SCIENCE 7. WORLD DIGITAL LIBIRARY
1.CHENNAI STUDENTS 2.BANGLORE STUDENTS 3. CURRENT AFFAIRS
CONTACT FOR ADVERTISEMENT IN ABOVE CHANNLES
ADMIN1: ADMIN2:
SHANKAR TM

IAS ACADEMY
^ The Best IAS Academy In South India SINCE 2004

31 35
32. 35
33. 36
34. 37
35 . 38
36 . 40
37. 41
38 . 41
39 . 42
40 . 43
41. 44
42. .................................................................................................................................45
43. 46
44. 48
45 . 49
46 . 50
47. 50
48 . 51
49 . 52
50. 53
51. 53
52. 54
53. 55
54. 56
55 . ................................................................................................................................. 57
56 . 57
57. 58
58 . 61
59 . 61
60. 62
61. 62
62. 63
63. 64
64. 65
65 . 66
66 . .................................................................................................................................67
67. 68
68. 69
69 . 69
70. 70

2
SHANKAR TM

IAS ACADEMY
^ The Best IAS Academy In South India SINCE 2004

71. 71
72. 72
73. 72
74. 73
75. 74
76. 75
77. 76
78. 78
79 . 78
80. 79
81. 80
82. 80
83. 83
84. 85
85. 86
86 . 87
87. 88
88. 88
89 . 89
90. 90
91. 91
92. 91
93. 93
94. 94
95. 94
96. 96
97. 96
98. 97
99 . 98
100. 99

3
SHANKAR TM

IAS ACADEMY
^ The Best IAS Academy In South India SINCE 2004

PRESTORMING 2020
MODERN INDIA II - EXPLANATIONS

1 . Consider the following Passage:


“An Indian State where northern and southern parts hilly , while the central part has fertile
plains. Rice is a major crop that is intensively cultivated in this State . It has less
population density than the national average and most of its population lives in its rural
areas ( 76%) . This State does have better performance in the Sex Ratio ( 991) but poor
female literacy rate ( 60%). Also , it has less road density and railway density . Nearly 42% of
the State is under forest cover. It has 3 National Parks , 11 Wildlife Sanctuaries and
IBiosphere Reserve based on UNESCO’s MAB Program. Both Moist and Dry Deciduous
forest can be found this State .”

Which of the following species can be found in their natural habitat in the State described
in the above passage?

1. Asiatic lion

2. Royal Bengal Tiger


3. Snow Leopard

4. One -horned Rhinoceros


5 . Olive Ridley Turtle
6. Wild Water Buffalo

Select the correct answer using the code given below:

( a ) 2 and 6 only
( b) 3 and 4 only

(c) 1, 5 and 6 only


(d ) 2 , 4 and 5 only
EXPLANATION:

State - Chhattisgarh

( Even without knowing the State, answer can be found using the clues given in the passage)

 Asiatic lion is found only in Gujarat. Gujarat does not have hills in both northern and
southern part with central fertile plains.
 Snow Leopard is primarily found in the states of Jammu & Kashmir , Himachal Pradesh ,
Uttarakhand , Sikkim and Arunachal Pradesh . But , from the passage , the State must be a
Tropics. Also they are hilly states while doesn 't match with the description of central fertile
palin .
 One- Horned Rhinos are throughout the entire stretch of the Indo- Gangetic Plain. But these

4
SHANKAR TM

IAS ACADEMY
^ The Best IAS Academy In South India SINCE 2004

areas have higher population density .


By this elimination , option (a ) can be arrived .

( Or by use of UNESCO World Heritage Site , answer can be arrived )


This is a tropical state. So, mostly it will be Kerala , Tamil Nadu ( Does not have hills in North and
South . Kerala have hills in East and plains in the west ) , Madhya Pradesh ( have highest area under
forest cover but not high forest density ) , Chhattisgarh and West Bengal ( high population density) .
So , Option ( a ) is correct .

2. Consider the following statements about coastlines:


1 . No other country has a long coastline on the Indian Ocean like India .
2. Andhra Pradesh state has the longest mainland coastline in India .
Which of the statements given above is / are correct?

(a ) 1 only
( b) 2 only
(c ) Both 1 and 2
( d ) Neither 1 nor 2
EXPLANATION:

No other country has a long coastline on the Indian Ocean as India has and indeed , it is
India’s eminent position in the Indian Ocean which justifies the naming of an Ocean after
it . So , Statement 1 is correct .

Gujarat has the longest mainland coastline in India. So , Statement 2 is not correct .

3. With reference to Tropic of Cancer In India , consider the following statements:


1 . Most of the geographical area of Chhattisgarh is located North of Tropic of
Cancer.
2. The Tropic of cancer does not pass through Rajasthan .
Which of the statements given above is / are correct?

(a) 1 only
( b) 2 only
(c ) Both 1 and 2
( d ) Neither 1 nor 2

5
SHANKAR TM

IAS ACADEMY
^ The Best IAS Academy In South India SINCE 2004

EXPLANATION:

The most of the geographical area of the Chhattisgarh is located on the southern side of
Tropic of Cancer.So , Statement 1 is not correct .
States through which the Tropic of Cancer passes.
36°N V
l- INDIA
/
LX '
1
f &Z ADJACENT COUNTRIES 36°N
9
JAMMU
r
32° )
AFGHANISTAN
r* j
AND
KASHMIR j /
s 32°
9

-fRADCEH
-
H
j CHINA
n K~ PAKISTAN / fWMJ iJ UTTM . * ( T I B E T )
) J - &
'H RWM1 V

r . j Vu2 *>.
Sx RAJASTHAN
4

24°
.
'A
CALCAR AT©
- •

0
^^
PRVOESM
^ M A
/•
20°

AHASIAN
-
I
*
TVS? BtSMtHMU
- 41
JVflWi

, MAHARASHTRA i f/f - •
OOiSHA

j*s
HWMsnfck

BAY OF
MYANMA* 2< f

. r• i/
*

IfP HEA . *

*» L ‘V f BENGAL

The Tropic of cancer passes through Rajasthan .So, Statement 2 is not correct.

States through which the Tropic of Cancer passes.


-/ 3 !•
k

36°N X I INDIA
y LX \ 9
•A
ADJACENT COUNTRIES 36°N

! AFGHANISTAN < JAMMU


AND /
32° ) r* ( KASHMIR *

s #
32°
J ’fflADCEHi
K~ *4*^;® UTTwS?
4 Vv
CHINA

PAKISTAN
r - "V ( T I B E T )

.
) t . -
' HljRWMl

Vu '* -
< 001 * 4 -
s
J
r *
XMiit
p
RAJABTHAN
x
24° * ** V B ! H A R '7
$,«• • *> vj 24°
* C.ART
.
20° •r ’"'
’ y HJvMxai h
JWAU COSHA IlMai 20°

AH A S I A N
MMU
MAHARASHTRA rv . • ;j S * BAY OF
SEA •* Y/ X\ vr ' BENGAL
.
°
1A

4. Which of the following plate(s) is / are parts of Gondwana Land?


1. South American plate
2. African plate

6
SHANKAR TM

IAS ACADEMY
k \ The Best IAS Academy In South India SINCE 2004

3 . Indo - Australian plate


4. Antarctic plate
Select the correct answer using the code given below:

(a) 3 only
( b) 1 and 2 only
(c ) 1 , 2 and 4 only
( d ) 1 , 2, 3 and 4

EXPLANATION:

AUSTRALIA 7.
A
ANTARCTICA /

^0?

r-H CokJ water currents
f H Warm waiei cunents
\Aouritains
S Land To animat «d map
/

V
I!>.jL
1 \ X

to -
Shallow sea? of all geologic
[ 1Deep ocean basins time periods

South American , African , Indo-Australian and Antarctic plate was part of Gondwana
land . So , option ( d ) is correct .

ADDITIONAL INFORMATION:

Gondwana, also called Gondwanaland , ancient supercontinent that incorporated present-


day South America, Africa, Arabia , Madagascar , India, Australia, and Antarctica . It was
fully assembled by Late Precambrian time , some 600 million years ago , and the first stage
of its breakup began in the Early Jurassic Period , about 180 million years ago. The name
Gondwanaland was coined by the Austrian geologist Eduard Suess in reference to
Upper Paleozoic and Mesozoic formations in the Gondwana region of central India, which
are similar to formations of the same age on Southern Hemisphere continents.

7
SHANKAR TM

IAS ACADEMY
^ The Best IAS Academy In South India SINCE 2004

According to plate tectonic evidence , Gondwana was assembled by continental collisions in


the Late Precambrian (about 1 billion to 542 million years ago ) . Gondwana then collided
with North America, Europe , and Siberia to form the supercontinent of Pangea . The
breakup of Gondwana occurred in stages. Some 180 million years ago , in the Jurassic
Period , the western half of Gondwana (Africa and South America) separated from the
eastern half ( Madagascar , India, Australia , and Antarctica) . The South Atlantic Ocean
opened about 140 million years ago as Africa separated from South America . At about the
same time , India, which was still attached to Madagascar , separated from Antarctica and
Australia , opening the central Indian Ocean . During the Late Cretaceous Period , India
broke away from Madagascar , and Australia slowly rifted away from Antarctica. India
eventually collided with Eurasia some 50 million years ago , forming the Himalayan
mountains, while the northward- moving Australian plate had just begun its collision along

the southern margin of Southeast Asia a collision that is still under way today .

5. Consider the following statements:


1 . The land masses of Peninsular Plateau are more unstable in nature compared
to the land masses of Himalayan region .
2. The Peninsular Plateau is composed of igneous and metamorphic rocks with
steep hills and narrow valleys.
Which of the statements given above is / are correct?

(a) 1 only
( b) 2 only
(c ) Both 1 and 2
( d ) Neither 1 nor 2
EXPLANATION:

Geologically , the Peninsular Plateau constitutes one of the ancient landmasses on the
earth’s surface . It was supposed to be one of the most stable land blocks. So , Statement
1 is not correct .
The Peninsular plateau is a tableland composed of the old crystalline , igneous and
metamorphic rocks. But Penisular plateau is in old stage and hence , it has broad valley
( Narrow valley is the characteristics of youth stage ) . So , Statement 2 is not correct .

ADDITIONAL INFORMATION:

The Peninsular plateau is a tableland composed of the old crystalline , igneous and
metamorphic rocks. It was formed due to the breaking and drifting of the Gondwana land
8
SHANKAR TM

IAS ACADEMY
^ The Best IAS Academy In South India SINCE 2004

and thus, making it a part of the oldest landmass . The plateau has broad and shallow
valleys and rounded hills . This plateau consists of two broad divisions , namely , the Central
Highlands and the Deccan Plateau . The part of the Peninsular plateau lying to the north of
the Narmada river covering a major area of the Malwa plateau is known as the Central
Highlands . The Vindhyan range is bounded by the Central Highlands on the south and the
Aravalis on the northwest.

SOURCE:

Geography NCERT

6 . Consider the following statements about the Himalayan mountains:


1 . The altitude of Himalayas is greater in the West than those in the East .

2. The folds of Greater Himalayas are symmetrical in nature .


3. The Himalayan hills running through the north-eastern states are mostly
composed of strong sandstones .
Which of the statements given above is / are correct?

(a) 1 only
(b ) 1 and 3 only
(c ) 2 and 3 only
( d ) 1 , 2 and 3
EXPLANATION:

The altitude of Himalayas is greater in the West than in the East . But , altitudinal
variations are greater in Eastern half than those in the Western half . So , Statement 1 is
correct .

The folds of Greater Himalayas are asymmetrical in nature . Himalayas were formed
mainly by the movement of Indian plate from Gondwana land towards the Eurasian plate
which leads to the subduction of Tethys sea . This movement of indian plate continues
and around 25 million years these two plates ( Eurasian and Indian plates) touches each
other and due to further movement with a STRONG FORCE gives rise to middle
Himalayas or lower Himalayas with a large number of asymmetrical folds as a
discontinuous range due to Nappe structure between them . So , Statement 2 is not
correct .

Beyond the Dihang gorge , the Himalayas bend sharply towards south and form
the Eastern hills or Purvachal. These hills run through the north eastern states of India.
They are mostly composed of sandstones. These hills are composed ofthe Patkai Hills,
Naga Hills, Manipuri Hills and Mizo Hills. So , Statement 3 is correct .

9
SHANKAR TM

IAS ACADEMY
4 The Best IAS Academy In South India SINCE 2004

7. Consider the following statements about regarding the Gravitational Waves .

1. Gravitational waves are invisible and they are incredibly fast .


2. The most powerful gravitational waves are created when objects move at very
slow speeds .
3. LIGO is the world 's largest gravitational wave observatory .

Which of the statements given above are correct?

(a) 1 and 3 only

( b) 1 and 2 only

(c) 2 and 3 only


(d ) 1 , 2 and 3

EXPLANATION:

Gravitational waves are invisible . However , they are incredibly fast . They travel at
the speed of light ( 186 , 000 miles per second ) . Gravitational waves squeeze and stretch
anything in their path as they pass by . So , Statement 1 is correct .

The most powerful gravitational waves are created when objects move at
very high speeds . So , Statement 2 is not correct .

LIGO is the world ' s largest gravitational wave observatory. It consists of two detectors
situated 1 ,865 miles (3,002 kilometers) apart in isolated regions in the states of
Washington and Louisiana .So , Statement 3 is correct .

ADDITIONAL INFORMATION:

A gravitational wave is an invisible (yet incredibly fast ) ripple in space .


We’ve known about gravitational waves for a long time . More than 100 years ago , a
great scientist named Albert Einstein came up with many ideas about gravity and
space .
Einstein predicted that something special happens when two bodies such as —
planets or stars—orbit each other. He believed that this kind of movement could
cause ripples in space . These ripples would spread out like the ripples in a pond
when a stone is tossed in . Scientists call these ripples of space gravitational waves .
Gravitational waves are invisible . However, they are incredibly fast . They travel at the
speed of light ( 186 , 000 miles per second ) . Gravitational waves squeeze and stretch
anything in their path as they pass by.

10
SHANKAR TM

IAS ACADEMY
4 The Best IAS Academy In South India SINCE 2004

What causes gravitational waves?


The most powerful gravitational waves are created when objects move at very high
speeds. Some examples of events that could cause a gravitational wave are:
 when a star explodes asymmetrically ( called a supernova)
 when two big stars orbit each other
 when two black holes orbit each other and merge
But these types of objects that create gravitational waves are far away. And
sometimes , these events only cause small , weak gravitational waves . The waves are
then very weak by the time they reach Earth . This makes gravitational waves hard to
detect.

SOURCE:

https: / / spaceplace .nasa.gov / gravitational-waves / en /

8. The Amazon rainforest is spread in which of the following countries?


1. Brazil
2. Peru
3. Colombia
4. Chile

Select the correct answer using the code given below .

(a) 1 and 2 only


( b) 1 , 2 and 3 only
(c) 1 , 2 and 4 only
(d ) 1 , 2 , 3 a n d 4

EXPLANATION:

The Amazon Rainforest is spread across 9 South American countries, so there ' s
many options . Brazil , Peru , Colombia, Venezuela , Ecuador , Bolivia, Guyana,
Suriname and French Guiana are all a part of the rainforest .

9. Consider the following statements about the Eco -Sensitive Zones which are
frequently in news:
1. These are the areas within 10 kilometres around Protected Areas , National
Parks and Wildlife Sanctuaries .

11
SHANKAR TM

IAS ACADEMY
^ The Best IAS Academy In South India SINCE 2004

2. These are declared under the Environment Protection Act , 1986 by the
Ministry of Environment , Forest and Climate Change .
3. Within these zones , rainwater harvesting and organic farming are permitted .

Which of the statements given above are correct?

(a) 1 and 3 only


( b) 1 and 2 only
(c) 2 and 3 only
(d ) 1 , 2 and 3

EXPLANATION:

Section 9 of the Wildlife Conservation Strategy 2002 states “ Lands falling within 10 kms
of the boundaries of National Parks and Sanctuaries should be notified as eco-fragile
zones under section 3(2 ) (v ) of the Environment ( Protection ) Act and Rule 5 Sub- rule 5(viii)
& (x ) of the Environment (Protection ) Rules” . The Supreme Court in its order dated
December 4 , 2006 gave specific directions on declaration of an area of 10 km around
Protected Areas as ESAs . So , Statement 1 is correct .
Ecologically Sensitive Areas (ESAs) have been identified and notified by the Indian
Ministry of Environment & Forests ( MoEF) since 1989 . Notifications declaring areas as
ESAs are issued under the Environment ( Protection ) Act 1986 . So , Statement 2 is
correct .
Permitted activities are ongoing agricultural or horticultural practices, rainwater
harvesting, organic farming , use of renewable energy sources, adoption of green
technology for all activities . So , Statement 3 is correct .

SOURCE:
http : / / assets .wwfindia . org / downloads / indias_ notified_ecologicallysensitive_areas . pdf
10. Consider the following statements about the Bioplastics .
1 . Bioplastic is a biodegradable material that comes only from renewable sources .
2. Unlike conventional plastics , the Bio Plastic items can be broken down by
microbes in industrial composting facilities .
3. Net contribution to negative environment impact like Eutrophication and
Acidification potentials are higher for conventional plastics than Bioplastics .
Which of the statements given above is / are correct?

(a) 1 only
( b) 2 only
(c) 2 and 3 only
(d ) 1 , 2 and 3
12
SHANKAR TM

IAS ACADEMY
^ The Best IAS Academy In South India SINCE 2004

EXPLANATION:

Bioplastics are biodegradable materials that come from renewable sources and can be
used to reduce the problem of plastic waste that is suffocating the planet and
contaminating the environment . Bioplastic can be made from agricultural by-
products and also from used plastic bottles and other containers using microorganisms.
Common plastics , such as fossil-fuel plastics (also called petrobased polymers) are
derived from petroleum or natural gas . Not all bioplastics are biodegradable nor
biodegrade more readily than commodity fossil-fuel derived plastics . So , Statement 1 is
not correct .
Unlike conventional plastics, the Bio Plastic items can be broken down by microbes in
industrial composting facilities. So , Statement 2 is correct .
Bioplastics induce higher eutrophication potentials than conventional plastics . So ,
Statement 3 is not correct .

ADDITIONAL INFORMATION:

As an alternative , the use of bioplastics is being promoted , consisting in obtaining


natural polymers from agricultural , cellulose or potato and corn starch waste .
These are 100% degradable , equally resistant and versatile , already used
in agriculture , textile industry , medicine and , over all , in the container and
packaging market, and biopolymers are already becoming popular in cities
throughout Europe and the United States for ecological reasons: they are known as
PHA .
This product is expected to cover the needs of 10% of the European plastics market
within 10 years.
PHA as bioplastic
These are polyesters produced by fermenting raw vegetable materials with a series of
bacterial strains . For example , PHAs can be used for injection molding to build
automobile parts and for many other uses . Specifically , PHA ( polyhydroxyalkanoate )
is extracted from bacteria such as pseudomonas. In its natural form , it is similar to
transparent kitchen film, with the difference that it is an authentic bioplastic.
Advantages of bioplastics:
 They reduce carbon footprint
 They providing energy savings in production
 They do not involve the consumption of non - renewable raw materials
 Their production reduces non - biodegradable waste that contaminates the
environment
 They do not contain additives that are harmful to health , such as phthalates or
bisphenol A
 They do not change the flavor or scent of the food contained

13
SHANKAR TM

IAS ACADEMY
4 The Best IAS Academy In South India SINCE 2004

1 1. Consider the following pairs:

Himalayan division Name

1. Between Indus and Sutluj Kumaon Himalaya

2. Between Tista and Kali Assam Himalaya

3. Between Dihang and Tista Nepal Himalaya

Which of the pairs given above is / are not correctly matched?


(a) 1 only
( b) 1 and 2 only
(c ) 2 and 3 only
( d ) 1 , 2 and 3
EXPLANATION:

The part of the Himalayas lying between Satluj and Kali rivers is known as Kumaon
Himalayas. So , Pair 1 is not correctly matched .
The Kali and Tista rivers demarcate the Nepal Himalayas and the part lying between
Tista and Dihang rivers is known as Assam Himalayas . So , Pairs 2 and 3 are not
correctly matched .

ADDITIONAL INFORMATION:

The Himalayas have been divided on the basis of regions from west to east . These divisions
have been demarcated by river valleys . For example , the part of Himalayas lying between
Indus and Satluj has been traditionally known as Punjab Himalaya but it is also known
regionally as Kashmir and Himachal Himalaya from west to east respectively . The part of
the Himalayas lying between Satluj and Kali rivers is known as Kumaon Himalayas . The
Kali and Tista rivers demarcate the Nepal Himalayas and the part lying between Tista and
Dihang rivers is known as Assam Himalayas . There are regional names also in these broad
categories.

SOURCE:

Geography NCERT

12. With reference to Northern Plains of India , consider the following statements:
1 . The Northern Plains are generally characterized as flat land with no variations
in relief.

2. The largest portion of Northern Plain is formed of older alluvium .


14
SHANKAR TM

IAS ACADEMY
4 The Best IAS Academy In South India SINCE 2004

Which of the statements given above is / are correct?

(a) 1 only
(b ) 2 only
(c ) Both 1 and 2
( d ) Neither 1 nor 2
EXPLANATION:

The northern plains are generally described as flat land with no variations in its relief . It
is not true . These vast plains also have diverse relief features . So , Statement 1 is not
correct .
The largest part of the northern plain is formed of older alluvium . They lie above the flood
plains of the rivers and present a terrace like feature . This part is known as Bhangar . So ,
Statement 2 is correct .

ADDITIONAL INFORMATION:

The Northern plains can be divided into four regions. The rivers , after descending from the
mountains deposit pebbles in a narrow belt of about 8 to 16 km in width lying parallel to
the slopes of the Shiwaliks . It is known as bhabar . All the streams disappear in this bhabar
belt . South of this belt , the streams and rivers re -emerge and create a wet, swampy and
marshy region known as terai . This was a thickly forested region full of wildlife . The largest
part of the northern plain is formed of older alluvium . They lie above the flood plains of the
rivers and present a terrace like feature . This part is known as bhangar . The soil in this
region contains calcareous deposits locally known as kankar. The newer , younger deposits
of the flood plains are called khadar . They are renewed almost every year and so are fertile ,
thus, ideal for intensive agriculture .

SOURCE:

Geography NCERT

13. Which among the following rivers is not flowing in the East?
(a) Pennar
( b) Brahmani

(c ) Kalinadi
( d ) Palar

15
SHANKAR TM

IAS ACADEMY
^ The Best IAS Academy In South India SINCE 2004

EXPLANATION:

The Kalinadi rises from Belgaum district and falls in the Karwar Bay. It’s a small west
flowing river while Pennar , brahmani and Palar are small east flowing rivers. So , option
(c ) is correct .

ADDITIONAL INFORMATION:

West flowing small rivers:

Riocr Catchment area


sq . km
Sabai mat i 21,674
MahA 34,642
Dhandtiar 2,770
Kalinadi 5, 179
Sharavati 2,029
Bharathapuzha 5,397
Perlyar 5,243

East flowing small rivers:

River Catchment arm


sq . km
Submiarekha 19,296
Batlaml 12, 789
Brahman! 39,033
Penner 55,213
Palar 17,670

SOURCE:

Geography NCERT

14. With reference to the Island groups of India , consider the following statements:
1 . Saddle peak in the South Andaman is the highest mountain peak in the
Andaman and Nicobar islands .
2. The Pittilsland is located in the Lakshadweep .
3. 11 DegreeChannel separates the Andaman and Nicobar Islands while 9 Degree
Channel separates Luccadive Islands and Minicoy Island .
Which of the statements given above is / are correct?

(a ) 2 only
( b) 1 and 3 only
(c ) 2 and 3 only
( d ) 1, 2 and 3

16
SHANKAR TM

IAS ACADEMY
^ The Best IAS Academy In South India SINCE 2004

EXPLANATION:

Saddle Peak is located on North Andaman Islands . At 731 m it is the highest point of the
archipelago in the Bay of Bengal . So , Statement lis not correct .
Pitti Island is uninhabited coral islet in Lakshadweep . It has been declared a bird
sanctuary and is even one of the most famous bird watching sites in India owing to the
fact that nearly 75 sea birds species can be found here . So , Statement 2 is correct .
The Ten Degree Channel separates the Andaman Islands and Nicobar Islands from each
other in the Bay of Bengal . So , Statement 3 is not correct .

SOURCE:

https: / / www.britannica.com / place / Andaman - Islands# ref 18081

15 . Consider the following statements about Pradhan Mantri Jan Arogya Yojana
( PM JAY ) :
1 . It provides cover of Rs . 5 lakhs per family per year , for secondary and tertiary
care hospitalization across public and private empanelled hospitals in India .
2. There is a cap on family size and age of members for the eligibility to this
scheme .
3. It provides cashless access to health care services for the beneficiary at the
point of service , that is , the hospital.
4. Benefits of this scheme are portable across the country .

Select the correct answer using the code given below .

(a) 1 , 2 and 3 only


( b) 1 , 2 and 4 only
(c) 1 , 3 and 4 only
(d ) 2 , 3 and 4 only

EXPLANATION:

The second component under Ayushman Bharat is PM-JAY , which aims at providing
health insurance cover of Rs . 5 lakhs per family per year for secondary and tertiary care
hospitalization to over 10.74 crores poor and vulnerable families (approximately 50 crore
beneficiaries ) . So , Statement 1 is correct .
There is no cap on the family size under the scheme . This scheme was earlier known as
National Health Protection Scheme ( NHPS ) before it was rechristened to PM-JAY. So ,

17
SHANKAR TM

IAS ACADEMY
^ The Best IAS Academy In South India SINCE 2004

Statement 2 is not correct .


PM -JAY provides cashless access to health care services for the beneficiary at the point of
service , that is , the hospital. So , Statement 3 is correct .
Benefits of the scheme are portable across the country i. e . a beneficiary can visit any
empanelled public or private hospital for cashless treatment. So , Statement 4 is
correct .

ADDITIONAL INFORMATION:

Pradhan Mantri Jan Arogya Yojana ( PM-JAY )

The second component under Ayushman Bharat is PM-JAY , which aims at providing health
insurance cover of Rs . 5 lakhs per family per year for secondary and tertiary care
hospitalization to over 10.74 crores poor and vulnerable families (approximately 50 crore
beneficiaries ) . There is no cap on the family size under the scheme . This scheme was earlier
known as National Health Protection Scheme ( NHPS) before it was rechristened to PM-JAY.
This scheme was launched on 23rd September 2018 by the HonT>le Prime Minister Shri
Narendra Modi in Ranchi , Jharkhand .

PM -JAY has been rolled out for the bottom 40% of poor and vulnerable population .
The households included are based on the deprivation and occupational criteria of
Socio- Economic Caste Census 2011 (SECC 2011) for rural and urban areas
respectively. The scheme subsumed then existing Rashtriya Swasthya Bima Yojana
( RSBY ) , launched in 2008 . Therefore , the coverage mentioned under PM-JAY also
includes families that were covered in RSBY but were not present in the SECC 2011
database . PM -JAY is completely funded by the Government , and cost of
implementation is shared between Central and State Governments.
Key Features of PM -JAY
 PM -JAY is the world ’s largest health insurance / assurance scheme fully financed by
the government .
 PM -JAY provides cover of Rs. 5 lakhs per family per year , for secondary and tertiary
care hospitalization across public and private empaneled hospitals in India.
 Over 10.74 crore poor and vulnerable entitled families (approximately 50 crore
beneficiaries) are eligible for these benefits.
 PM -JAY provides cashless access to health care services for the beneficiary at the
point of service , that is, the hospital.
 PM -JAY will help reduce catastrophic expenditure for hospitalizations , which pushes
6 crore people into poverty each year , and will help mitigate the financial risk arising
out of catastrophic health episodes.
 No restrictions on family size , age or gender.
18
SHANKAR TM

IAS ACADEMY
^ The Best IAS Academy In South India SINCE 2004

 All pre-existing conditions are covered from day one .


 Covers up to 3 days of pre- hospitalization and 15 days post- hospitalization expenses
such as diagnostics and medicines
 Benefits of the scheme are portable across the country i.e . a beneficiary can visit any
empanelled public or private hospital for cashless treatment.
 Services include approximately 1,393 procedures covering all the costs related to
treatment , including but not limited to drugs, supplies , diagnostic services,
physician ' s fees , room charges, surgeon charges, OT and ICU charges etc.
 Public hospitals are reimbursed for the healthcare services at par with the private
hospitals .

SOURCE:

https: / / www.pmjay.gov .in / about- pmjay

16 . Consider the following statements about the Motor Vehicle Act , 2019:
1. Offence committed by Juveniles is to be tried under Juveniles Justice Act and
not under this Act .
2. If a contractor of the road fails to comply with road design standards , then
there will be penalty on the road contractor.
3. If a vehicle manufacturer fails to comply with motor vehicle standards , then
there will be penalty on the Vehicle manufacturer.
4. The central government will develop a scheme for cashless treatment of road
accident victims during golden hour.
Which of the statements given above are correct?

(a) 2 and 3 only


( b) 1 , 2 and 3 only
(c ) 1 and 4 only
( d ) 1 , 2, 3 and 4
EXPLANATION:

Offence committed by Juveniles are to be tried under Juveniles Justice Act not under this
Act. So , Statement 1 is correct .
If a contractor fails to comply with road design standards, the penalty will be a fine of up
to one lakh rupees. So , Statement 2 is correct .
If a vehicle manufacturer fails to comply with motor vehicle standards, the penalty will be
a fine of up to Rs 100 crore , or imprisonment of up to one year , or both . So , Statement 3
is correct .

19
SHANKAR TM

IAS ACADEMY
4 The Best IAS Academy In South India SINCE 2004

The central government will develop a scheme for cashless treatment of road accident
victims during golden hour . The Bill defines golden hour as the time period of up to one
hour following a traumatic injury , during which the likelihood of preventing death through
prompt medical care is the highest. The central government may also make a scheme for
providing interim relief to claimants seeking compensation under third party insurance.
So , Statement 4 is correct .

ADDITIONAL INFORMATION:

The Motor Vehicles (Amendment ) Bill , 2019

The Motor Vehicles (Amendment) Bill, 2019 was introduced in Lok Sabha on July 15, 2019
by the Minister for Road Transport and Highways , Mr. Nitin Gadkari. The Bill seeks to
amend the Motor Vehicles Act , 1988 to provide for road safety. The Act provides for grant
of licenses and permits related to motor vehicles , standards for motor vehicles , and
penalties for violation of these provisions.

 Compensation for road accident victims: The central government will develop a
scheme for cashless treatment of road accident victims during golden hour. The Bill
defines golden hour as the time period of up to one hour following a traumatic
injury , during which the likelihood of preventing death through prompt medical care
is the highest. The central government may also make a scheme for providing
interim relief to claimants seeking compensation under third party insurance . The
Bill increases the minimum compensation for hit and run cases as follows: (i) in case
of death , from Rs 25 , 000 to two lakh rupees, and (ii) in case of grievous injury , from
Rs 12 ,500 to Rs 50 ,000 .
 Compulsory insurance: The Bill requires the central government to constitute a
Motor Vehicle Accident Fund , to provide compulsory insurance cover to all road
users in India. It will be utilised for: (i) treatment of persons injured in road
accidents as per the golden hour scheme , (ii) compensation to representatives of a
person who died in a hit and run accident , (iii) compensation to a person grievously
hurt in a hit and run accident , and (iv) compensation to any other persons as
prescribed by the central government. This Fund will be credited through: (i)
payment of a nature notified by the central government , (ii) a grant or loan made by
the central government , (iii) balance of the Solatium Fund (existing fund under the
Act to provide compensation for hit and run accidents) , or (iv) any other source as
prescribed the central government.
 Good Samaritans: The Bill defines a good Samaritan as a person who renders
emergency medical or non- medical assistance to a victim at the scene of an

20
SHANKAR TM

IAS ACADEMY
^ The Best IAS Academy In South India SINCE 2004

accident . The assistance must have been (i) in good faith , (ii) voluntary , and (iii)
without the expectation of any reward . Such a person will not be liable for any civil
or criminal action for any injury to or death of an accident victim , caused due to
their negligence in providing assistance to the victim .
 Recall of vehicles: The Bill allows the central government to order for recall of motor
vehicles if a defect in the vehicle may cause damage to the environment , or the
driver , or other road users. The manufacturer of the recalled vehicle will be required
to: (i) reimburse the buyers for the full cost of the vehicle , or (ii) replace the defective
vehicle with another vehicle with similar or better specifications .
 National Transportation Policy: The central government may develop a National
Transportation Policy , in consultation with state governments. The Policy will: (i)
establish a planning framework for road transport , (ii) develop a framework for grant
of permits , and (iii) specify priorities for the transport system , among other things.
 Road Safety Board: The Bill provides for a National Road Safety Board , to be created
by the central government through a notification . The Board will advise the central
and state governments on all aspects of road safety and traffic management
including: (i) standards of motor vehicles , (ii) registration and licensing of vehicles ,
(iii) standards for road safety , and (iv ) promotion of new vehicle technology.
 Offences and penalties: The Bill increases penalties for several offences under the
Act. For example , the maximum penalty for driving under the influence of alcohol or
drugs has been increased from Rs 2 , 000 to Rs 10 , 000 . If a vehicle manufacturer
fails to comply with motor vehicle standards, the penalty will be a fine of up to Rs
100 crore , or imprisonment of up to one year , or both . If a contractor fails to comply
with road design standards, the penalty will be a fine of up to one lakh rupees. The
central government may increase fines mentioned under the Act every year by up to
10%.
 Taxi aggregators: The Bill defines aggregators as digital intermediaries or market
places which can be used by passengers to connect with a driver for transportation
purposes (taxi services ) . These aggregators will be issued licenses by state Further ,
they must comply with the Information Technology Act , 2000 .
It is well known that India is one of the most accident-prone countries in the world ,
accounting for nearly 1 , 50 , 000 deaths, 10% of all motor vehicles- related fatalities
worldwide . According to the 2018 report of the World Health Organization , the
highest number of road accidents occur in India worldwide . Even China , the most
populous country , is behind us in this regard . As per the report of the Ministry of
Road Transport and Highways , 2017; there are about 5 lakh road
accidents occurred in India every year in which around 1.5 lakh people are killed .
There are around 1.49 lakh people died in 2018 in the road accidents with Uttar
Pradesh registering the maximum spike in fatalities. So in order to prevent the
menace of road accidents; the central government has amended the Motor Vehicle
21
SHANKAR TM

IAS ACADEMY
4 The Best IAS Academy In South India SINCE 2004

1988 by the Motor Vehicles (Amendment) Bill 2019 . Motor Vehicles (Amendment)
Act 2019 has been implemented throughout the country since September 1 , 2019 .
Now the penalty has been increased 10 times on various violations .

17 . Which of the following methods could be useful for the production of


Hydrogen?
1. Thermochemical processes
2. Electrolytic processes
3. Direct Solar Water splitting processes
4. Biological processes

Select the correct answer using the code given below .

(a) 1 and 3 only


( b) 1 , 2 and 3 only
(c) 1 and 2 only
( d ) 1 , 2 , 3 and 4

EXPLANATION:

Hydrogen can be produced using a number of different processes . Thermochemical


processes use heat and chemical reactions to release hydrogen from organic
materials such as fossil fuels and biomass. Water ( H20) can be split into hydrogen
( H 2 ) and oxygen (02) using electrolysis or solar energy . Microorganisms such as
bacteria and algae can produce hydrogen through biological processes .
Thermochemical Processes
Some thermal processes use the energy in various resources , such as natural gas,
coal , or biomass , to release hydrogen from their molecular structure . In other
processes, heat , in combination with closed -chemical cycles , produces hydrogen
from feedstocks such as water. Learn more about the following thermochemical
processes:
 Natural gas reforming (also called steam methane reforming or SMR )
 Coal gasification
 Biomass gasification
 Biomass- derived liquid reforming
 Solar thermochemical hydrogen (STCH ) .
Electrolytic Processes

22
SHANKAR TM

IAS ACADEMY
4 The Best IAS Academy In South India SINCE 2004

Electrolyzers use electricity to split water into hydrogen and oxygen . This technology
is well developed and available commercially , and systems that can efficiently use
intermittent renewable power are being developed . Learn more about electrolysis.
Direct Solar Water Splitting Processes
Direct solar water splitting , or photolytic , processes use light energy to split water
into hydrogen and oxygen . These processes are currently in the very early stages of
research but offer long- term potential for sustainable hydrogen production with low
environmental impact. Learn more about the following solar water splitting
processes:
 Photoelectrochemical ( PEC )
 Photobiological .
Biological Processes
Microbes such as bacteria and microalgae can produce hydrogen through biological
reactions , using sunlight or organic matter . These technology pathways are at an
early stage of research , but in the long term have the potential for sustainable , low-
carbon hydrogen production . Learn more about the following biological processes:
 Microbial biomass conversion
 Photobiological .

18 . Consider the following statements about the Protection of Children from


Sexual Offences ( POCSO ) Act , 2012:
1. On the lines of Juvenile Justice Act of 2015 this Act also defines a child as any
person below 14 years of age .
2. The Act seeks to protect children from offences such as sexual assault , sexual
harassment and pornography .
3. It is only applicable to child survivors and adult offenders and not to the cases
between two children or in the case of a child perpetrates a sexual offence on
an adult .
Which of the statements given above are correct?

(a) 1 and 3 only


( b) 1 and 2 only
(c) 2 and 3 only
(d ) 1 , 2 and 3

EXPLANATION:

The said POSCO Act defines a child as any person below eighteen years of age . So ,
Statement 1 is not correct .
The Act for the first time , defines “ penetrative sexual assault” , “sexual assault ” and

23
SHANKAR TM

IAS ACADEMY
4 The Best IAS Academy In South India SINCE 2004

“sexual harassment ” . Thus the The Act seeks to protect children from offences such as
sexual assault , sexual harassment , and pornography. So , Statement 2 is correct .
The POCSO Act is only applicable to child survivors and adult offenders . In case two
children have sexual relations with each other , or in case a child perpetrates a sexual
offence on an adult , the Juvenile Justice (Care and Protection of Children ) Act , 2000 , will
apply. So , Statement 3 is correct .

19 . Which among the following is a fresh water lake?


(a) Pan gong Tso
( b) Chilika Lake
(c ) Pachpadra Lake
( d ) Wular lake
EXPLANATION:

Wular Lake is one of the largest fresh water lakes in Asia . It is in Jammu and Kashmir .
The lake basin was formed as a result of tectonic activity. Jhelum River flows through
Srinagar and the Wular lake before entering Pakistan through a deep narrow gorge . So ,
option ( d ) is correct .

ADDITIONAL INFORMATION:

 Pangong Tso (Tibetan : "high grassland lake ") is an endorheic lake in the Himalayas
situated at a height of about 4 ,350 m . It is 134 km long and extends from India to
the Tibetan Autonomous Region , China .
 Chilika Lake is a brackish water lagoon in Odisha state on the east coast of India .
 The Pachpadra Lake is a salt lake near Pachpadra in Barmer District , Rajasthan .

SOURCE:

Geography NCERT

20. Which one of the following condition is referred as ‘Continentality’?


(a ) Very hot during summers and very cold in winters
( b) Moderately hot in summers and moderately cold in winters
(c ) Very cold during summers and very hot in winters
(d ) Moderate cold in summers and moderate hot in winters

24
SHANKAR TM

IAS ACADEMY
^ The Best IAS Academy In South India SINCE 2004

EXPLANATION:

As the distance from the sea increases, its moderating influence decreases and the
people experience extreme weather conditions. This condition is known as Continentality
(i.e . very hot during summers and very cold during winters ) . So , option (a ) is correct .

SOURCE:

Geography NCERT

21. With reference to onset and withdrawal of Indian monsoon , consider the
following statements:
1 . The Monsoon , unlike Trade winds , is steady in nature .
2. Normally , the withdrawal of the Monsoon begins in North -western States of
India by early September.
3. The withdrawal from the southern half of Indian Peninsula is fairly slower than
northern half of Indian Peninsula .
Which of the statements given above is / are correct?

(a ) 2 only
( b) 3 only
(c ) 1 and 2 only
( d ) 1 and 3 only
EXPLANATION:

The Monsoon , unlike the trades, are not steady winds but are pulsating in nature ,
affected by different atmospheric conditions encountered by it , on its way over the warm
tropical seas . So , Statement 1 is not correct .
Withdrawal or the retreat of the monsoon is a more gradual process. The withdrawal of
the monsoon begins in North -Western states of India by early September . By mid-
October , it withdraws completely from the northern half of the peninsula . So , Statement
2 is correct .
The withdrawal from the southern half of the peninsula is fairly rapid . By early
December , the monsoon has withdrawn from the rest of the country. So , Statement 3 is
not correct .

ADDITIONAL INFORMATION:

The duration of the monsoon is between 100- 120 days from early June to mid -September.
Around the time of its arrival , the normal rainfall increases suddenly and continues

25
SHANKAR TM

IAS ACADEMY
4 The Best IAS Academy In South India SINCE 2004

constantly for several days . This is known as the hurst’ of the monsoon , and can be
distinguished from the pre - monsoon showers . The monsoon arrives at the southern tip of
the Indian peninsula generally by the first week of June . Subsequently , it proceeds into two
- the Arabian Sea branch and the Bay of Bengal branch . The Arabian Sea branch reaches

Mumbai about ten days later on approximately the 10 th of June . This is a fairly rapid
advance . The Bay of Bengal branch also advances rapidly and arrives in Assam in the first
week of June .

SOURCE:

http: / / ncert.nic.in / ncerts / l / iessl 04 . pdf

22. With reference to winter season in India , Consider the following statements:
1. The average temperature decreases from South to North of India steadily.
2 . The Southwest Trade wind prevail over the country during this season .
3. The Peninsular region does not have well defined cold season in India.
Which of the statements given above is / are correct ?

(a ) 3 only
( b) 1 and 2 only
(c ) 2 and 3 only
( d ) 1 , 2 and 3
EXPLANATION:

For example , Chennai and Nellore . Geographically, Nellore is located north of Chennai.
But in Winter , Nellore has higher temperature when compared to Chennai which have
coastal influence . So , it cannot be generalized that temperature will decrease from South
to North of India. But temperature gradually decreases from coastal to interior part of
India . So , Statement 1 is not correct .
During this season , the Northeast Trade Winds prevail over the country. So , Statement
2 is not correct .
The peninsular region does not have a well-defined cold season . There is hardly any
noticeable seasonal change in temperature pattern during winters due to the moderating
influence of the sea . So , Statement 3 is correct .

ADDITIONAL INFORMATION:

A characteristic feature of the cold weather season over the northern plains is the inflow of
cyclonic disturbances from the west and the northwest . These low- pressure systems,
26
SHANKAR TM

IAS ACADEMY
^ The Best IAS Academy In South India SINCE 2004

originate over the Mediterranean Sea and western Asia and move into India , along with the
westerly flow. They cause the much - needed winter rains over the plains and snowfall in the
mountains . Although the total amount of winter rainfall locally known as ‘mahawat’ is
small , they are of immense importance for the cultivation of ‘rabi’ crop .

SOURCE:

Geography NCERT

23. In general , which among the following have least impact on Indian Monsoon?

(a) The shift in the position of Inter Tropical Convergence Zone in summer
( b) Intense heating of Tibetan plateau
(c ) Presence of high pressure area to the east of Madagascar
( d ) Cyclonic formations in the Temperate Zone
EXPLANATION:

Cyclonic formation in the temperate zone does not have much impact on Indian
monsoon . So , Option ( d ) is correct .

ADDITIONAL INFORMATION:

The monsoons are experienced in the tropical area roughly between 20° N and 20 ° S . To
understand the mechanism of the monsoons , the following facts are important.

(a) The differential heating and cooling of land and water creates low pressure on the
landmass of India while the seas around experience comparatively high pressure .

( b ) The shift of the position of Inter Tropical Convergence Zone (ITCZ) in summer , over the
Ganga plain ( this is the equatorial trough normally positioned about 5° N of the equator . It
is also known as the monsoontrough during the monsoon season ) .

( c ) The presence of the high - pressure area, east of Madagascar , approximately at 20 ° S over
the Indian Ocean . The intensity and position of this high - pressure area affects the Indian
Monsoon .

( d ) The Tibetan plateau gets intensely heated during summer , which results in strong
vertical air currents and the formation of low pressure over the plateau at about 9 km
above sea level .

( e ) The movement of the westerly jet stream to the north of the Himalayas and the presence
of the tropical easterly jet stream over the Indian peninsula during summer .

SOURCE:
Geography NCERT
27
SHANKAR TM

IAS ACADEMY
^ The Best IAS Academy In South India SINCE 2004

24. With reference to the Indian Monsoon , consider the following statements:
1 . The monsoon winds cover the entire India in about a month .
2. Whenever the axis of monsoon trough shifts closer to the Himalayas , there is
longer and abundant rainfall in the Northern Plains of India .
Which of the statements given above is / are correct?

(a) 1 only
( b) 2 only
(c ) Both 1 and 2
( d ) Neither 1 nor 2
EXPLANATION:

The Southwest monsoon sets in over the Kerala coast by 1st June and moves swiftly to
reach Mumbai and Kolkata between 10 th and 13th June . By mid -July , southwest
monsoon engulfs the entire subcontinent . So , Statement 1 is not correct .
When the axis of the monsoon trough lies over the plains, rainfall is good in these parts.
On the other hand , whenever the axis shifts closer to the Himalayas , there are longer dry
spells in the plains , and widespread rain occur in the mountainous catchment areas of
the Himalayan rivers. So , Statement 2 is not correct .

ADDITIONAL INFORMATION:

A phenomenon associated with the monsoon is its tendency to have ‘breaks’ in rainfall.
Thus , it has wet and dry spells. In other words , the monsoon rains take place only for a few
days at a time . They are interspersed with rainless intervals. These breaks in monsoon are
related to the movement of the monsoon trough . For various reasons, the trough and its
axis keep on moving northward or southward , which determines the spatial distribution of
rainfall . When the axis of the monsoon trough lies over the plains , rainfall is good in these
parts . On the other hand , whenever the axis shifts closer to the Himalayas , there are longer
dry spells in the plains, and widespread rain occur in the mountainous catchment areas of
the Himalayan rivers. These heavy rains bring in their wake , devastating floods causing
damage to life and property in the plains . The frequency and intensity of tropical
depressions too , determine the amount and duration of monsoon rains. These depressions
form at the head of the Bay of Bengal and cross over to the mainland . The depressions
follow the axis of the “ monsoontrough of low pressure”. The monsoon is known for its
uncertainties . The alternation of dry and wet spells varies in intensity , frequency and
duration . While it causes heavy floods in one part , it may be responsible for droughts in the
other . It is often irregular in its arrival and its retreat.

28
SHANKAR TM

IAS ACADEMY
^ The Best IAS Academy In South India SINCE 2004

SOURCE:

Geography NCERT

25 . Consider the following statements about the Sulphur emissions:


1. India is considered as the largest emitter of the Sulphur dioxide in the World .
2. The primary reason for India’s high emission output is the expansion of coal -
based electricity generation .
3. The vast majority of Thermal power plants in India lack flue -gas desulfurization
technology to reduce their air pollution .
Which of the statements given above is / are correct?

(a) 1 only
( b) 1 and 2 only
(c) 2 and 3 only
(d ) 1 , 2 and 3

EXPLANATION:

According to a new analysis by Greenpeace , India is the largest emitter of S02 in the
world with more than 15% of all the anthropogenic sulphur dioxide (SO2 ) hotspots
detected by NASA OMI ( Ozone Monitoring Instrument ) satellite . So , Statement 1 is
correct .
The primary reason for India’s high emission output is the expansion of coal- based
electricity generation . So , Statement 2 is correct .
The vast majority of plants in India lack flue-gas desulfurization technology to reduce
their air pollution . So , Statement 3 is correct .

26 . Consider the following statements about the Green Rating Project:


1. It is an effort to rate industrial units within a specific sector on the basis of
their environment friendliness .
2. It is regarded as India’s only independent , rigorous and credible programme of
its kind.
3. It is prepared by the Centre for Science and Environment , a not -for-profit
public interest research and advocacy organisation .

Which of the statements given above is / are correct?

(a) 1 only
( b) 1 and 2 only
(c) 2 and 3 only
(d ) 1 , 2 and 3
29
SHANKAR TM

IAS ACADEMY
^ The Best IAS Academy In South India SINCE 2004

EXPLANATION:

The Green Rating Project ( GRP) is an effort to rate industrial units within a specific sector
on the basis of their environment friendliness. The project aims at encouraging companies
to adopt better environment management policies . Environment risk liability is an issue
that is gaining increasing attention in Indian company’s boardroom .So , Statement 1 is
correct .

GRP is a unique programme that rates Indian industry for its environmental performance .
It is regarded as India’s only independent , rigorous and credible programme of its kind . So ,
Statement 2 is correct .

It is prepared by the Centre for Science and Environment , a not-for- profit public interest
research and advocacy organisation . So , Statement 3 is correct .

SOURCE:

https: / / www. cseindia . org / about- green-rating- programme - 277

27. Which among the following is correct about ‘e -green watch’ which was recently
in news?
(a) It is a International Non - governmental organization related to conservation of forest
(b ) It is a portal related to Compensatory Afforestation Fund Management and
Planning Authority
(c ) It is a United Nations initiative to promote electronic mobility
( d ) It is a high resolution camera invented to survey the forest cover by the Forest
Survey of India
EXPLANATION:

Since the primary function of CAMPA is the regeneration of vegetation cover and
promoting afforestation as a way of compensating for forest land which is diverted to
non -forest uses . It , therefore , calls for an information system so as to collect and present
information to monitor and track how well CAMPA funds are achieving this end . In view
of the above background , e- Green Watch is being developed as an integrated & online
system that will be completely transparent , reliable and accountable . It will also present
the data in real time and shall be accessible to all stakeholders and public at large . It
will allow for monitoring, evaluation , social and ecological audits by independent
organizations researchers and the public. So , Option (b ) is correct .

30
SHANKAR TM

IAS ACADEMY
^ The Best IAS Academy In South India SINCE 2004

ADDITIONAL INFORMATION:

 Compensatory Afforestation Fund Management and Planning Authority ( CAMPA) are


meant to promote afforestation and regeneration activities as a way of compensating
for forest land diverted to non-forest uses .
 Due to certain discrepancies in the implementation of compensatory afforestation ,
some NGOs had approached The Honhle Supreme Court for relief. The HonTole
Supreme Court on 10th July 2009 issued orders that there will be a Compensatory
Afforestation Fund Management and Planning Authority ( CAMPA) as National
Advisory Council under the chairmanship of the Union Minister of Environment &
Forests for monitoring, technical assistance and evaluation of compensatory
afforestation activities.

SOURCE:

http: / / egreenwatch .nic .in / Public / About_Iccmes.aspx

28. If a ship travels along the Sea lines of Communication from Hong Kong to Abu
Dhabi , which of the following straits will be crossed by that ship assuming it
moves along the shortest possible route?
1. Hormuz Strait
2. Cook Strait
3. Gibraltar Strait
4. Malacca Strait

Select the correct answer using the code given below .

(a) 1 only
( b ) 1 and 4 only
(c) 2 and 4 only
(d ) 1, 3 and 4 only

31
SHANKAR TM

IAS ACADEMY
^ The Best IAS Academy In South India SINCE 2004

EXPLANATION:

Cape of Good Hope


5.8

29 . Consider the following statements about the State Rooftop Solar Attractiveness
Index ( SARAL) :
1. It is the first of its kind index to provide a comprehensive overview of state -
level measures adopted to facilitate rooftop solar deployment .
2. It has been launched by the Ministry of Power and the Ministry of New and
Renewable Energy .
3. Karnataka has been placed at the first rank in the Index followed by Telangana ,
Gujarat and Andhra Pradesh .

Which of the statements given above are correct?

(a) 1 and 2 only


( b) 1 and 3 only
(c) 2 and 3 only
(d ) 1 , 2 and 3
32
SHANKAR TM

IAS ACADEMY
^ The Best IAS Academy In South India SINCE 2004

EXPLANATION:

The MNRE has developed the State Rooftop Solar Attractiveness Index-SARAL that
evaluates Indian states based on their attractiveness for rooftop development . SARAL is
the first of its kind index to provide a comprehensive overview of state-level measures
adopted to facilitate rooftop solar deployment .So , Statement 1 is correct .

The Ministry of New and Renewable Energy ( MNRE) has launched the State Rooftop Solar
Attractiveness Index-SARAL.So , Statement 2 is correct .

Karnataka has been placed at the first rank in the index followed by Telangana, Gujarat
and Andhra Pradesh .So , Statement 3 is correct .

ADDITIONAL INFORMATION:

 SARAL is the first of its kind index to provide a comprehensive overview of state-level
measures adopted to facilitate rooftop solar deployment.
 It would create a more conducive environment for solar rooftop installations,
encourage investment and lead to the accelerated growth of the sector , by creating
healthy competition among the States .
 Karnataka has been placed at the first rank in the index followed by Telangana ,
Gujarat and Andhra Pradesh .
 SARAL currently captures five key aspects :

o Robustness of policy framework ,


o Implementation environment ,
o Investment climate ,
o Consumer experience ,
o Business ecosystem
 It has been designed collaboratively by the MNRE , Shakti Sustainable Energy
Foundation (SSEF) , Associated Chambers of Commerce and Industry of India
(ASSOCHAM ) and Ernst & Young (EY ) .
 SARAL is in concurrence with the Government’s target of installing 175 GW of
renewable energy capacity by 2022 , of which 100 GW of solar power is to be
operational by March 2022 , ( 40 out of 100 GW is expected to come from grid -
connected solar rooftops ) .
 The rooftop solar deployment will make power sector sustainable and viable (as the
cost of solar energy is reducing) and it will help to ensure 24 / 7 power supply to all
consumers .

33
SHANKAR TM

IAS ACADEMY
4 The Best IAS Academy In South India SINCE 2004

 Along with that , it will help in India in fulling its Intended Nationally Determined
Contributions (INDCs) highlighted at Paris climate deal.

SOURCE:

PIB

30 . Consider the following statements about to the India State of forest


report ( ISFR ) :
1 . It published annually by the Forest Survey of India .
2. The report does not cover the forests of Andaman and Nicobar Islands .
Which of the statements given above is / are correct?

(a) 1 only
( b) 2 only
(c ) Both 1 and 2
( d ) Neither 1 nor 2

EXPLANATION:

Forest Survey of India ( FSI) has been assessing the forest and tree resources of our
country on a biennial (taking place every other year) basis. The results of the assessment
are published in its biennial report titled “ India State of Forest Report (ISFR ) . The first
state of forest report was brought out in 1987.Since then FSI has published 15 such
reports. So , Statement 1 is not correct .
The report covers the forests of Andaman and Nicobar Islands . So , Statement 2 is not
correct .

ADDITIONAL INFORMATION:

The 15th State of Forest Report shows that India’s total forest cover increased by 0.94 per
cent , from 7 ,01,673 square kilometers to 7 ,08 , 273 square kilometers since its last
assessment in 2015. The biennial report is prepared by The Forest Survey of India ( FSI ) , a
body under MoEFCC , which conducts surveys and assessment of forest resources in the
country. The report contains information on forest cover , tree cover , mangrove cover ,
growing stock inside and outside the forest areas, carbon stock in India’s forests and forest
cover in different patch size classes . Special thematic information on forest cover such as
hill, tribal districts, and north eastern region has also been given separately in the report .
The report for the first time contains information on decadal change in water bodies in

34
SHANKAR TM

IAS ACADEMY
^ The Best IAS Academy In South India SINCE 2004

forest during 2005- 2015, forest fire , production of timber from outside forest , state wise
carbon stock in different forest types and density classes.

SOURCE:

http: / / fsi.nic.in / isfr 2017 / isfr-introduction-2017 . pdf

https: / / pib.gov.in / PressReleseDetail . aspx?PRID = 1520284

31. Consider the following statements about the Atlantic Meridional Overturning
Circulation which sometimes seen in the news recently:
1 . It is one of the Earth’s largest water circulation systems where ocean currents move
warm , salty water from the tropics to regions further north.
2 . Rising temperatures in the Indian Ocean can boost the Atlantic Meridional Overturning
Circulation.

Which of the statements given above is / are correct?

(a ) 1 only

( b) 2 only

(c) Both 1 and 2

(d ) Neither 1 nor 2

EXPLANATION:

Atlantic Meridional Overturning Current is the large system of ocean currents operating in the
Atlantic Ocean . It circulates the waters between the north and the south. So , Statement 1 is
correct .
Warming as a result of climate change, the Indian Ocean is causing a series of cascading effects
that is providing AMOC a “jump start”. So , Statement 2 is correct .

32. In India , the highest incidence of wastelands is recorded in which among the
following states?
(a ) Rajasthan
(b) Gujarat
(c) Uttar Pradesh
(d ) Telangana

35
SHANKAR TM

IAS ACADEMY
^ The Best IAS Academy In South India SINCE 2004

EXPLANATION:

Rajasthan tops the chart with 25% of its geographical area being wasteland , accounting
for 18% of the wasteland nationally. Accoring to the latest wasteland atlas , the wastelands
have undergone positive change in the states of Rajasthan (0.48 Mha) , Bihar (0.11 Mha) ,
Uttar Pradesh (0.10 Mha) , Andhra Pradesh (0.08 Mha) , Mizoram (0.057 Mha) , Madhya
Pradesh (0.039 Mha) , Jammu 85 Kashmir (0.038 Mha) and West Bengal (0.032 Mha) .So ,
option ( a ) is correct .

ADDITIONAL INFORMATION:

In an effort to productively use India’s wastelands , the ministry of rural development has
come out with the fifth edition of Wasteland Atlas - 2019 , eight years after the last edition
was published in 2011. This is significant as it takes into account 12.08 MHa of unmapped
area of Jammu and Kashmir (J & K ) for the first time .

The department of land resources in collaboration with National Remote Sensing Centre
( NRSC ) , Department of Space has published Wastelands Atlases of India - 2000 , 2005 ,
2010 8s 2011 editions . The new wastelands mapping exercise , carried out by NRSC using
the Indian Remote Sensing Satellite data is the fifth edition of Wastelands Atlas 2019 .

SOURCE:

https: / / economictimes .indiatimes . com / news / economy / agriculture / ministry- of- rural-
development- releases-fifth -edition-of- wasteland -
atlas / articleshow / 71920250.cms?from = mdr

33. Consider the following statements about Alluvial soils in India:


1 . They are generally rich in organic matter and nitrogen but deficient in potash .
2. The drainage of the soil is poor and it lacks in profile development .
3. In Northern Plains, the sand content of the soil decreases from west to east .
Which of the statements given above is / are correct?

(a ) 3 only
( b) 2 and 3 only
(c ) 1 and 2 only
( d ) 1 , 2 and 3
EXPLANATION:

The alluvial soils vary in nature from sandy loam to clay. They are generally rich in
potash but poor in phosphorous . So , Statement 1 is not correct .

36
SHANKAR TM

IAS ACADEMY
^ The Best IAS Academy In South India SINCE 2004

Alluvial soils are depositional soil - transported and deposited by rivers , streams etc.
They are well drained . So , Statement 2 is not correct .
The sand content of alluvial soil decreases from west to east as the carry capacity of the
river decreases with decrease in elevation . So river loses capacity to carry bigger particles
and deposits it and carries only finer particles such as silt and clay. So , statement 3 is
correct .

ADDITIONAL INFORMATION:

 The alluvial soil consists of various proportions of sand , silt and clay. As we move
inlands towards the river valleys , soil particles appear some what bigger in size . In
the upper reaches of the river valley i.e . near the place of the break of slope , the soils
are coarse . Such soils are more common in piedmont plains such as Duars , Chos
and Terai . Apart from the size of their grains or components , soils are also described
on the basis of their age . According to their age alluvial soils can be classified as old
alluvial ( Bangar ) and new alluvial ( Khadar ) . The hangar soil has higher concentration
of kanker nodules than the Khadar. It has more fine particles and is more fertile
than the bangar.
 Alluvial soils as a whole are very fertile . Mostly these soils contain adequate
proportion of potash , phosphoric acid and lime which are ideal for the growth of
sugarcane , paddy , wheat and other cereal and pulse crops. Due to its high fertility ,
regions of alluvial soils are intensively cultivated and densely populated . Soils in the
drier areas are more alkaline and can be productive after proper treatment and
irrigation .

SOURCE:

Geography NCERT

34. Consider the following pairs:

Soil Major crop grown

1. Laterite soil Cashew nut


2. Red soil groundnut
3. Black soil Cotton
Which of the pairs given above is / are correctly matched?
(a) 3 only
( b) 1 and 2 only
(c ) 2 and 3 only
( d ) 1 , 2 and 3

37
SHANKAR TM

IAS ACADEMY
^ The Best IAS Academy In South India SINCE 2004

EXPLANATION:

Laterite soils in Tamil Nadu , Andhra Pradesh and Kerala are more suitable for crops like
cashew nut. So , Pair 1 is correctly matched.
In tropical and subtropical conditions of India , groundnut is extensively grown in textured
red sandy , aluuvial and coastal alluvial soils as well as red soils. So , Pair 2 is correctly
matched.
Black soil is ideal for growing cotton and is also known as black cotton soil . So , Pair 3 is
correctly matched .

ADDITIONAL INFORMATION:

Red soil develops on crystalline igneous rocks in areas of low rainfall in the eastern and
southern part of the Deccan Plateau . Along the piedmont zone of the Western Ghat , long
stretch of area is occupied by red loamy soil. Yellow and red soils are also found in parts of
Odisha and Chattisgarh and in the southern parts of the middle Ganga plain . The soil
develops a reddish colour due to a wide diffusion of iron in crystalline and metamorphic
rocks. It looks yellow when it occurs in a hydrated form . The fine - grained red and yellow
soils are normally fertile , whereas coarse -grained soils found in dry upland areas are poor
in fertility. They are generally poor in nitrogen , phosphorous and humus .

SOURCE:

Geography NCERT

35 . Consider the following statements about the Nutrient Based Subsidy Policy in
India:
1. It is a policy under the Ministry of Agriculture and Farmers Welfare .
2. Under this , a fixed amount of subsidy decided on an annual basis is provided on
each grade of subsidized Phosphatic & Potassic ( P&K ) fertilizers .
3. Fertilizer subsidy is the largest subsidy to exchequer followed by Food and Fuel
subsidy .
Which of the statements given above are not correct?

(a) 1 and 2 only


( b) 1 and 3 only
(c ) 2 and 3 only
( d ) 1 , 2 and 3
EXPLANATION:

It is being implemented from April 2010 by the Department of Fertilizers , Ministry of


38
SHANKAR TM

IAS ACADEMY
4 The Best IAS Academy In South India SINCE 2004

Chemicals & Fertilizers .So , Statement 1 is not correct .

Under NBS , a fixed amount of subsidy decided on an annual basis , is provided on each
grade of subsidized Phosphatic & Potassic ( P&K ) fertilizers depending on its nutrient
content .So , Statement 2 is correct .

Fertilizer subsidy is the second- biggest subsidy after food subsidy , the inaction on the part
of the government is not only worrying for the fiscal health of the economy but also
detrimental to the soil health of the country .So , Statement 3 is not correct .

ADDITIONAL INFORMATION:

 Government provides fertilizers , Urea and 21 grades of P&K fertilizers to farmers at


subsidized prices through fertilizer manufacturers / importers.
 In accordance to its farmer friendly approach , the Govt is committed to ensure the
availability of P&K fertilizers to farmers on affordable price .
 The subsidy on P&K fertilizers is being governed by NBS Scheme from 2010 .
The Nutrient Based Subsidy ( NBS) Scheme
 It is being implemented from April 2010 by the Department of Fertilizers, Ministry of
Chemicals & Fertilizers.
 Under NBS , a fixed amount of subsidy decided on an annual basis , is provided on
each grade of subsidized Phosphatic & Potassic ( P&K) fertilizers depending on its
nutrient content .
NBS Analysis
 The NBS scheme , was sought to deregulate subsidy on non-urea fertilizers and
expected to reduce the subsidy burden substantially.
 While the NBS certainly did not lead to any decline in subsidy on fertilizer , it did lead
to worsening of soil nutrient quality , along with shortages and price increases in all
three types of major nutrients , namely Nitrogenous , Phosphoric and Potassic.
 An undesirable outcome has been the change in fertilizer mix. As against the
recommended Nitrogen ( N ) : Phosphorous ( P) : Potassium ( K ) ratio of 4:2:1, the NPK
ratio in 2013- 14 was 8.2:3.2:1.
 The environmental damage caused by the inappropriate use of fertilizers is certainly
a matter of serious concern in many states.
 Considering that fertilizer subsidy is the second-biggest subsidy after food subsidy ,
the inaction on the part of the government is not only worrying for the fiscal
health of the economy but also detrimental to the soil health of the country.
 Since fertilizer prices follow the trend in international petroleum prices , the only way
to reduce the subsidy bill is to reduce the dependence on imports and increase
domestic production .
39
SHANKAR TM

IAS ACADEMY
4 The Best IAS Academy In South India SINCE 2004

 While rationalizing fertilizer subsidy across nutrients may be the short- term and
immediate solution to the problem , the need of the hour is to have a policy
framework that incentivizes domestic production of fertilizers .
 The bigger challenge is to change the pattern of fertilizer use . This not only
involves revamping and re-energizing the extension services but also changing the
NBS suitably to remove the price distortion caused by it .

SOURCE:

PIB

36 . Consider the following statements about the Otter species:


found all over the world except in Australia , New Zealand ,
1. Otters are
Madagascar , and other Oceanic Islands .
2. India is home to Eurasian Otter , Smooth-coated Otterand Small-clawed otter of
the 13 species of otters found worldwide .
3. All Otter species are included in Appendix I of the Convention on International
Trade in Endangered Species ( CITES ) .
Which of the statements given above are correct?

(a) 1 and 2 only


( b) 1 and 3 only
(c) 2 and 3 only
(d ) 1 , 2 and 3

EXPLANATION:

Otters are found the world over, except in Australia, New Zealand , Madagascar , and other
oceanic islands. So , Statement 1 is correct .
India is home to 3 of the 13 species of otters found worldwide . These are -

Eurasian Otter ( Lutra lutra) ; Smooth -coated Otter ( Lutra perspicillata) and Small-
clawed otter (Aonyx cinereus ) . So , Statement 2 is correct .
Only two otters , asian small-clawed and smooth- coated otters were included in Appendix I
of the Convention on International Trade in Endangered Species ( CITES ) at the ongoing
18th Conference of Parties ( CoP) in Geneva on August 26 , 2019 . So , Statement 3 is not
correct .

SOURCE:

https: / / www.downtoearth . org.in / news / wildlife - biodiversity / cites-cop- 2019 -otters- given -
highest - protection-from - trade-66360

40
SHANKAR TM

IAS ACADEMY
4 The Best IAS Academy In South India SINCE 2004

37. The feature ‘Parkland landscape’ is specific to which among the following types
of forest?
(a) Moist deciduous forest
(b ) Dry deciduous forest
(c ) Swamp forest
( d ) Tropical semi-evergreen forest
EXPLANATION:

In the higher rainfall regions of the Peninsular plateau and the northern Indian plain ,
tropical dry deciduous forests have a parkland landscape with open stretches in which
teak and other trees interspersed with patches of grass are common . So , Option ( b ) is
correct .

ADDITIONAL INFORMATION:

Dry deciduous forest covers vast areas of the country, where rainfall ranges between 70 -
100 cm . On the wetter margins , it has a transition to the moist deciduous , while on the
drier margins to thorn forests. These forests are found in rainier areas of the Peninsula and
the plains of Uttar Pradesh and Bihar. In the higher rainfall regions of the Peninsular
plateau and the northern Indian plain , these forests have a parkland landscape with open
stretches in which teak and other trees interspersed with patches of grass are common . As
the dry season begins , the trees shed their leaves completely and the forest appears like a
vast grassland with naked trees all around . Tendu , palas, amaltas , bel , khair , axlewood ,
etc. are the common trees of these forests . In the western and southern part of Rajasthan ,
vegetation cover is very scanty due to low rainfall and overgrazing .

SOURCE:

Geography NCERT

38. Consider the following statements about the National Crisis Management
Committee :
1. It is one of the newly constituted permanent Cabinet Committee in India .
2. It is headed by the Prime Minister of India .
3. It is setup in the wake of a natural calamity for effective coordination and
implementation of relief measures and operations .
Which of the statements given above are not correct?

(a) 1 and 2 only


41
SHANKAR TM

IAS ACADEMY
^ The Best IAS Academy In South India SINCE 2004

( b) 1 and 3 only
(c) 2 and 3 only
(d ) 1 , 2 and 3

EXPLANATION:

A National Crisis Management Committee is a committee set up by the Government of


India in the wake of a natural calamity for effective coordination and implementation
of relief measures and operations . It is a Adhoc committee . So , Statement 1 is not
correct .
Composition:
 Cabinet Secretary (Chairperson) .
 Secretaries of Ministries / Departments and agencies with specific Disaster
management responsibilities .
So , Statement 2 is not correct .
It is setup in the wake of a natural calamity for effective coordination and implementation
of relief measures and operations.
So , Statement 3 is correct .

SOURCE:

http: / / www. disastermgmt . bih . nic.in / National% 20 Policy / Management% 20 Committee . htm

39 . Consider the following statements about Tropical evergreen forests in India:


1 . These forest found in the western slope of Western Ghats and hills of the
north -eastern region of India .
2. Species found in these forest include Rosewood , Hurra , Amaltas , Aini and
Ebony.
Which of the statements given above is / are correct?

(a) 1 only
( b) 2 only
(c ) Both 1 and 2
( d ) Neither 1 nor 2
EXPLANATION:

Tropical Evergreen Forests are also called tropical rainforests . These thick forests
occur in the regions near the equator and close to the tropics . These forests are found
in the western slope of the Western Ghats, hills of the northeastern region and the

42
SHANKAR TM

IAS ACADEMY
4 The Best IAS Academy In South India SINCE 2004

Andaman and Nicobar Islands . So , Statement 1 is correct .


Species found in these forests include Rosewood , Mahogony , Aini , Ebony , Hurra ,
Amaltas , etc . So , Statement 2 is correct .

ADDITIONAL INFORMATION:

Tropical evergreen forests are found in warm and humid areas with an annual precipitation
of over 200 cm and mean annual temperature above 22 oC . Tropical evergreen forests are
well stratified , with layers closer to the ground and are covered with shrubs and creepers ,
with short structured trees followed by tall variety of trees . In these forests, trees reach
great heights up to 60 m or above . There is no definite time for trees to shed their leaves ,
flowering and fruition .

SOURCE:

Geography NCERT

40 . Which among the following statements about the Beresheet mission , recently
seen in news, is correct?
(a) It is a mission of China to explore the water in the Moon
( b) It was a demonstrator of a small robotic lunar lander and lunar probe operated
by the Israel Aerospace Industries
(c) It is India’s anti- satellite mission
(d ) It is a mission of NASA to cross the heliosphere

EXPLANATION:

Beresheet , which means "in the beginning , " was a private mission to the Moon by
Israeli non - profit SpacelL. Built to win the now- defunct $ 20 million Google Lunar
XPrize , Beresheet was meant to inspire more Israelis to pursue STEM careers .
In addition to providing high - resolution imagery from the surface , Beresheet would
have measured the magnetic field at its landing site in Mare Serenitatis, which has
magnetic anomalies detected by Kaguya, Lunar Prospector , and the Luna 21
mission . Understanding the Moon ' s magnetism teaches us about its history. While
Earth has a global magnetic field caused by the continued churning of liquid metal
near the core , the Moon does not . But 3.6 billion years ago , the Moon had a
magnetic field just as strong as Earth ' s.

SOURCE:

https: / / www.planetary. org / explore / space- topics / space - missions / beresheet . html

43
SHANKAR TM

IAS ACADEMY
4 The Best IAS Academy In South India SINCE 2004

41 . Consider the following statements about Montane forests in Himalaya:


1 . Deodar , a highly valued endemic species grows mainly in the eastern part of
the Himalayan range .
2. The southern slope of the Himalaya has thicker vegetation cover than the
northern slope .
Which of the statements given above is / are correct?

(a) 1 only
(b ) 2 only
(c ) Both 1 and 2
( d ) Neither 1 nor 2
EXPLANATION:

Deodar , a highly valued endemic species grows mainly in the western part of the
Himalayan range . Deodar is a durable wood mainly used in construction activity. So ,
Statement 1 is not correct .
The southern slopes of the Himalayas carry a thicker vegetation cover because of
relatively higher precipitation than the drier north -facing slopes. So , Statement 2 is
correct .

ADDITIONAL INFORMATION:

 In mountainous areas , the decrease in temperature with increasing altitude leads to


a corresponding change in natural vegetation . Mountain forests can be classified
into two types, the northern mountain forests and the southern mountain forests .
The Himalayan ranges show a succession of vegetation from the tropical to the
tundra , which change in with the altitude . Deciduous forests are found in the
foothills of the Himalayas. It is succeeded by the wet temperate type of forests
between an altitude of 1 , 000- 2 ,000 m .
 In the higher hill ranges of northeastern India, hilly areas of West Bengal and
Uttaranchal , evergreen broad leaf trees such as oak and chestnut are predominant.
Between 1 ,500- 1 ,750 m , pine forests are also well- developed in this zone , with Chir
Pine as a very useful commercial tree . Deodar , a highly valued endemic species
grows mainly in the western part of the Himalayan range . Deodar is a durable wood
mainly used in construction activity . Similarly , the chinar and the walnut , which
sustain the famous Kashmir handicrafts, belong to this zone . Blue pine and spruce
appear at altitudes of 2 ,225-3,048 m . At many places in this zone , temperate
grasslands are also found . But in the higher reaches there is a transition to Alpine
forests and pastures . Silver firs , junipers, pines, birch and rhododendrons , etc.

44
SHANKAR TM

IAS ACADEMY
4 The Best IAS Academy In South India SINCE 2004

occur between 3, 000 - 4 , 000 m . However , these pastures are used extensively for
transhumance by tribes like the Gujjars , the Bakarwals , the Bhotiyas and the
Gaddis.
 The southern slopes of the Himalayas carry a thicker vegetation cover because of
relatively higher precipitation than the drier north -facing slopes. At higher altitudes,
mosses and lichens form part of the tundra vegetation . The southern mountain
forests include the forests found in three distinct areas of Peninsular India viz; the
Western Ghats , the Vindhyas and the Nilgiris . As they are closer to the tropics , and
only 1 ,500 m above the sea level , vegetation is temperate in the higher regions , and
subtropical on the lower regions of the Western Ghats, especially in Kerala, Tamil
Nadu and Karnataka . The temperate forests are called Sholas in the Nilgiris ,
Anaimalai and Palani hills. Some of the other trees of this forest of economic
significance include , magnolia, laurel , cinchona and wattle . Such forests are also
found in the Satpura and the Maikal ranges .

SOURCE:

Geography NCERT

42. Which among the following biosphere reserves is not a part of UNESCO’s ‘Man
and Biosphere Programme’?
(a) Gulf of Mannar
( b) Nanda Devi
(c ) Manas
( d ) Similipal
EXPLANATION:

The UNESCO has introduced the designation ‘Biosphere Reserve’ ( BR) for natural areas to
minimize conflict between development and conservation . BRs are nominated by national
government which meet a minimal set of criteria and adhere to minimal set of conditions
for inclusion in the world network of Biosphere reserves under the Man and Biosphere
Reserve Programme of UNESCO . Globally , there are 686 biosphere reserves in 122
countries , including 20 transboundary sites .
India has 11 internationally recognised BRs. They are as follows.
1. Nilgiri
2 . Gulf of Mannar
3. Sunderban
4. Nanda Devi
5. Nokrek
6 . Pachmarhi

45
SHANKAR TM

IAS ACADEMY
^ The Best IAS Academy In South India SINCE 2004

7. Similipal
8. Achanakmar-Amarkantak
9 . Great Nicobar
10 . Agasthyamala
11. Khangchendzonga
So , option ( c ) is the correct .

ADDITIONAL INFORMATION:

The Biosphere Reserve Programme is guided by UNESCO Man and Biosphere ( MAB)
programme as India is a signatory to the landscape approach supported by MAB
programme . A scheme called Biosphere Reserve is being implemented by Government of
India since 1986 , in which financial assistance is given in 90 : 10 ratio to the North Eastern
Region States and three Himalayan states and in the ratio of 60 : 40 to other states for
maintenance , improvement and development of certain items . The State Government
prepares the Management Action Plan which is approved and monitored by Central MAB
Committee .

SOURCE:

http: / / vikaspedia .in / energy / environment / biodiversity- 1 / biosphere - reserves- in -


india # section-3

43. What is common to places known as Dumas Ubharat , Dhamra and Kazhuveli?
(a) Tropical dry deciduous forest
(b ) Mangrove forest
(c ) Fresh water lakes
( d ) Tropical thorn forests
EXPLANATION:

Dumas Ubharat , Dhamra and Kazhuveli are all known for mangrove forests in India . So ,
option (b ) is correct .

ADDITIONAL INFORMATION:

Mangrove forests of India are globally unique with the highest record of biodiversity, gifted
with the mangrove genetic paradise at Bhitarkanika , and the globally threatened wildlife
species in the Sundarbans . The Sundarbans of India and Bangladesh is the only largest
mangrove forest in the world colonized by the Royal Bengal Tigers . Mangroves are dense
and floristically diverse along the east coast of India and the Andaman and Nicobar Islands .

46
SHANKAR TM

IAS ACADEMY
4 The Best IAS Academy In South India SINCE 2004

They are largely distributed in the high energy tidal coast of two extreme conditions: (i)
humid and wet in Sundarbans with rich biodiversity , and (ii) arid and dry in Gujarat with
low biodiversity. Despite increasing pressures , the mangrove cover in India increases
annually at the rate of 1.2% , as against the global mangrove cover that disappears at
0.66% . However , India has a large track of sparse mangrove stand .

Mangrove Sites State/Union Territories


Sunderbans West Bengal

Bhaitarkanika Orissa
Mahanadi
Subamarekha
Devi-Kauda
Dhamra
Mangrove Genetic Resources Centre
Chilka

Coringa East Andhra Pradesh


Godavari
Krishna

North Andaman Andaman & Nicobar


Nicobar

Pichavaram Tamil Nadu


Muthupet
Ramnad

Pulicat
Kaznuveli

Vembanad Kerala
Kannur (North Kerala)

Coondapur Karnataka
Dakshin Kannada / Hannavar
Karwar
Mangalore Forest Division

Goa Goa

Achra- Ratnagiri Maharashtra


Dev garh-Vijay
Durg
Veldur

47
SHANKAR TM

IAS ACADEMY
^ The Best IAS Academy In South India SINCE 2004

ADDITIONAL INFORMATION:

Density of population is defined as the number of persons per square kilometer . It helps in
getting a better understanding of the spatial distribution of population in relation to land .
The density of population in India ( 2011) is 382 persons per sq km . There has been a
steady increase of more than 200 persons per sq km over the last 50 years as the density of
population increased from 117 persons / sq km in 1951 to 382 persons / sqkm in 2011.

SOURCE:

Geography NCERT

http: / / censusindia.gov.in / 201 l - prov- results / data_files / india / Final_ PPT_201 lchapter7 . pdf

45 . Consider the following statements about the Neglected Tropical Diseases


( NTDs ) .
1 . These a diverse group of communicable diseases that prevail in tropical and
subtropical conditions .
2. They are caused by a variety of pathogens such as Viruses , Bacteria , Protozoa
and Helminths .
3. HIV / AIDS , Tuberculosis , and Malaria are not considered as the NTDs .
Which of the statements given above are correct?

(a) 1 and 2 only


( b) 1 and 3 only
(c) 2 and 3 only
(d ) 1 , 2 and 3

EXPLANATION:

Neglected tropical diseases ( NTDs )- a diverse group of communicable diseases that prevail
in tropical and subtropical conditions in 149 countries affect more than one billion
people and cost developing economies billions of dollars every year .So , Statement 1 is
correct .

NTDs have little in common , being caused by a variety of pathogens , ranging from viruses
to bacteria, fungi , protozoa, and helminths . So , Statement 2 is correct .

The neglected tropical diseases ( NTDs ) are an ever- growing list of infections that
predominate in the tropics and are neglected in comparison with the “ big three” : malaria,
tuberculosis (TB ) , and HIV / AIDS. So , Statement 3 is not correct .

49
SHANKAR TM

IAS ACADEMY
^ The Best IAS Academy In South India SINCE 2004

SOURCE:

https: / / www.ncbi.nlm . nih . gov / books / NBK62509 /

46 . Consider the following statements about the International Coalition for


Disaster Resilient Infrastructure ( CDRI ) :
1. It aims to bring developed and developing countries together to build synergies
to reach the goal of disaster-resilient infrastructure .
2. It was launched by India at the UN Climate Action Summit , 2019 held in New
York , USA.
3. It is an international coalition of countries, United Nations ( UN ) agencies ,
multilateral development banks , the private sector and academic institutions .
Which of the statements given above is / are correct?

(a) 1 only
( b) 1 and 2 only
(c) 2 and 3 only
(d ) 1 , 2 and 3

EXPLANATION:

CDRI brings together developed and developing countries to build synergies to reach the
goal of disaster- resilient infrastructure .So , Statement 1 is correct .

The Prime Minister of India, Narendra Modi , launched the Coalition for Disaster- Resilient
Infrastructure ( CDRI ) at the UN Secretary- General’s Climate Action Summit in New York ,
US . So , Statement 2 is correct .

It is an international coalition of countries , United Nations ( UN ) agencies , multilateral


development banks, the private sector , and academic institutions . So , Statement 3 is
correct .

SOURCE:

https: / / sdg.iisd . org / news / india-launches- global-coalition -for-disaster- resilient-


infrastructure /

47 . Consider the following statements about theComposite Water Management


Index ( CWMI ):
1. It is an important tool to assess and improve the performance of Indian States /
Union Territories in efficient management of water resources .
2. The National Institute for Transforming India ( NITI ) Aayog has developed the
Composite Water Management Index ( CWMI ) .

50
SHANKAR TM

IAS ACADEMY
4 The Best IAS Academy In South India SINCE 2004

3. Gujarat is ranked first and is followed by Andhra Pradesh , Madhya Pradesh ,


Goa , Karnataka and Tamil Nadu.
Which of the statements given above is / are correct?

(a) 1 only
( b) 1 and 2 only
(c ) 2 and 3 only
( d ) 1 , 2 and 3
EXPLANATION:

The CWMI is an important tool to assess and improve the performance of states and
union territories in efficient management of water resources . So , Statement 1 is correct .

The National Institute for Transforming India ( NITI ) Aayog has developed the Composite
Water Management Index (CWMI) to enable effective water management in Indian states
in the face of this growing crisis. So , Statement 2 is correct .

Gujarat is the highest performer , closely followed by other high performers such as
Madhya Pradesh and Andhra Pradesh . So , Statement 3 is correct .

SOURCE:

https: / / niti.gov.in / sites / default / files / 2019-08 / CWMI - 2.0-latest . pdf

48 . According to the Ministry of New and Renewable Energy , which of the following
kinds of energy would be considered as Renewable Energy in India?
1 . Tidal Energy
2. Wave Energy
3. Ocean Thermal Energy Conversion ( OTEC)
4. Biomass Energy
Select the correct answer using the code given below .

(a) 1 and 2 only


( b) 1 , 2 and 4 only
(c) 1 , 2 and 3 only
(d ) 1 , 2 , 3 and 4

EXPLANATION:

Renewable energy is derived from natural processes that are replenished constantly such
as solar , wind , ocean , hydropower , biomass , geothermal resources, and biofuels and
hydrogen .
 Oceans cover 70 percent of the earth’s surface and represent an enormous amount

51
SHANKAR TM

IAS ACADEMY
^ The Best IAS Academy In South India SINCE 2004

of energy . Although currently under- utilised , Ocean energy is mostly exploited by


just a few technologies: Wave , Tidal , Current Energy and Ocean Thermal Energy.
 Biomass is an important source of energy accounting for about one third of the
total fuel used in our country and in about 90% of the rural households .
 The widespread use of biomass is for household cooking and heating. The types of
biomass used are agricultural waste , wood , charcoal or dried dung.
So , Option ( d ) is correct .

ADDITIONAL INFORMATION:

Coal , Oil and Natural gas are the non - renewable sources of energy. They are also called
fossil fuels as they are products of plants that lived thousands of years ago . Fossil fuels are
the predominantly used energy sources today. India is the third largest producer of coal in
the world , with estimated reserves of around 3, 19 , 020.33 million tonnes of Geological
Resources of Coal (as of 1.4 . 2018) . Coal supplies more than 70.87% of the country ' s total
production of energy by commercial sources . India consumes about 245 MT of crude oil
annually , and more than 70% of it is imported . Burning fossil fuels cause great amount of
environmental pollution .

SOURCE:

http: / / vikaspedia .in / energy / energy- basics / sources- of -energy


49 . With reference to mineral deposits in India , consider the following statements:
1 . Peninsular rocks contain most of the reserves of coal , metallic minerals , mica
but not non-metallic minerals .
2. Sedimentary rocks on the western and eastern flanks of the peninsula have
fossil fuel.
Which of the statements given above is / are correct?

(a) 1 only
(b ) 2 only
(c ) Both 1 and 2
( d ) Neither 1 nor 2

EXPLANATION:

Peninsular rocks contain most of the reserves of coal , metallic minerals, mica but also
non - metallic minerals . So , Statement 1 is not correct .

Sedimentary rocks on the western and eastern flanks of the peninsula, in Gujarat and

52
SHANKAR TM

IAS ACADEMY
^ The Best IAS Academy In South India SINCE 2004

Assam have most of the petroleum deposits. So , Statement 2 is correct .

SOURCE:

http: / / ncert .nic.in / ncerts / l / jess105. pdf

50 . In which among the following places Crude Oil reserves are found in India?

1 . Tamil Nadu
2. Andhra Pradesh
3. Gujarat
4. Rajasthan
5. Assam

Select the correct answer using the code given below .

(a) 3 and 5 only


( b) 1, 2 and 4 only
(c ) 3, 4 and 5 only
( d ) 1 , 2, 3 , 4 and 5

EXPLANATION:

The following table shows the estimated crude petroleum and natural gas reserves in India by state/region as on 31 March 201

Crude oil reserves Natural gas reserves


Region Share of oil (% ) Share of gas (% )
( in million metric tonnes) ( in BCM )

Arunachal Pradesh 1.52 0.25 0.93 0.07


Andhra Pradesh 8.15 1.35 48.31 3.75
Assam 159.96 26.48 158.57 12.29
Coal Bed Methane 0 0 106.58 8.26
Eastern Offshore ^ 40.67 6.73 507.76 39.37
Gujarat 118.61 19.63 62.28 4.83
Nagaland 2.38 0.39 0.09 0.01
Rajasthan 24.55 4.06 34.86 2.70
Tamil Nadu 9.00 1.49 31.98 2.48
Tripura 0.07 0.01 36.10 2.80
b]
Western Offshore 239.20 39.60 302.35 23.44
Total 604.10 100 1, 289.81 100

51. Consider the following sectors:


1 . Industry
2. Domestic

53
SHANKAR TM

IAS ACADEMY
^ The Best IAS Academy In South India SINCE 2004

3. Commercial
4. Agriculture
Which among the following is the correct order of the above sectors in terms of
increasing order of their electricity consumption?

( a ) 3 -4 -2 - 1
( b ) 1 - 2 - 4 -3
(c ) 1 - 4-3- 2
( d ) 2 -3-4- 1
EXPLANATION:

Fig 6.4: Consumption of Electricity by Sectors in


India during 2016- 17
Traction & Railways
2%
Others
Commercial
9%

Domestic
24%

Total consumption * 1066268

SOURCE:

http : / / mospi. nic.in / sites / default / files / publication_reports / Energy_Statistics_2018. pdf

52. Consider the following statements about Tropical Cyclones in India:


1 . Cyclones emerging in April- May are usually much stronger than those during
October- December
2. Cyclones are a frequent event in the eastern coast of India than the western
coast .
Which of the statements given above is / are correct?

(a) 1 only
(b ) 2 only
(c ) Both 1 and 2
( d ) Neither 1 nor 2

54
SHANKAR TM

IAS ACADEMY
^ The Best IAS Academy In South India SINCE 2004

EXPLANATION:

Cyclones emerging in April- May are usually much weaker than those during
October- December. So , Statement 1 is not correct .

Cyclones are a frequent event in the eastern coast of India compared to western
coast . So , Statement 2 is correct .

SOURCE:

https: / / www.iasparliament . com / current-affairs / geography- 1 / cyclone -fani- an - unusual-


storm

53. With reference to the geographic distribution of the estimated potential of


renewable energy , which among following States has the highest share in India?
(a) Tamil Nadu
( b) Gujarat
(c ) Rajasthan
( d ) Maharashtra
EXPLANATION:

The geographic distribution of the estimated potential of renewable power as on 31.03. 2016
reveals that Rajasthan has the highest share of about 14% ( 167276 MW) , followed by
Gujarat with 13% share ( 157158 MW ) and Maharashtra with 10% share ( 119893MW ) ,
mainly on account of solar power potential .

Fig 1.6: Statewise Estimated Potential of


Renewable Power in India as on 31.03. 2017
Andhra
Pradesh, 4.95
others, 16.93
Uttar
Pradesh, 2.70 12.17
"
\ Himachal
Telangana, 2.48 m| Pradesh, 3.74

Tamil Nadu, 5.40 Jammu &


Kashmir, 11.27

Karnataka, 8.68
Rajasthan, 16.21
Madhya
Odisha, 2.96 Pradcih, 7.43 Maharashtra, 5.0
6

SOURCE:

http : / / mospi. nic .in / sites / default / files / publication _reports / Energy_Statistics_ 2018. pdf

55
SHANKAR TM

IAS ACADEMY
4 The Best IAS Academy In South India SINCE 2004

54. Consider the following statements:


1 . India is critically deficient in the reserves and production of Bauxite .
2. The Balaghat mines in Madhya Pradesh are the leading producer of Manganese
in India .

Which of the statements given above is / are correct?


(a) 1 only
(h ) 2 only
(c ) Both 1 and 2
(d ) Neither 1 nor 2
EXPLANATION:

India is critically deficient in the reserve and production of copper and not bauxite . So ,
Statement 1 is not correct .

Balaghat Mine is the largest mine of the company which produces one of the best quality
of manganese ore in the country and is also the deepest underground manganese mine in
Asia . So , Statement 2 is correct .

ADDITIONAL INOFRMATION:

INDIA
IMf *OKTANT MINERAI S
PAKISTAN
1 3 2° -

CHINA
( T I II K T I
L.
^
r *
' /^
'

^
J •
BHUTAN \
-
'"-' •Vld* j.*
i
\
i
&»y«

Katn
AlU .
.
- rV H I
m
Hadanba
^ ’A
(^BANOLAOB
?
\
I
rX ^
V/ \
I
/ 24° -
r«opae olCamem

. l A
Nagpur > -
c BMnpur
* A Durg K«r*Ju)Kar \ M VANMAR
Br>and«ra MsukalS 20° -
-
<;r , joo «nur
A

a-
K Ofrtpul
A
VwrvafchafMtnwr
Raartagln A
16“
GOA
UarmtgM

-
IRON ORE FIELDS
Stwnogga A A IRON ORE M NES
KudrwmAti A A IRON ORE EXPORTING PORTS
Mangoh^u CNUiamagilimi MANGANESE
12*
BAUXITE 12*

* MICA
ANDAMAN A NCOBA* SLAAOS
INDIA|

K°N
INDIA,'
.
400 Uf

Sai
I°N -
72*fc “
76 LANKA H4® SI 2°I.

SOURCE:

http: / / ncert . nic.in / ncerts / l / jessl 05. pdf


56
FREE BOOKS, NOTES & VIDEOS FOR CIVILSERVICES

EBOOKS & UPSC PRELIMS USPC MAINS VIDEO FOR DAILY


MAGZINES MATERIALS MATERIALS CIVILSERVICES NEWSAPERS

SECUREIAS UPSC PRELIMS UPSC MAINS DELHI CIVILSERVICES


TESTSERIES TESTSERIES STUDENTS BOOKS

OPTIONAL SUBJECTS BOOKS, STATE PCS, SSC, BANKING


TEST SERIES, VIDEOS & NOTES BOOKS, TESTS VIDEOS & NOTES
1.GEOGRAPHY 1.UPPSC 2.SSC 3.MPSC
2.HISTORY 4.IBPS 5.RAS & RPSC
3.MATHEMATICS ENGINEERING BOOKS & MATERIAL
4. SOCIOLOGY 1. IES 2. GATE 3. IFoS
5.PUBLIC ADMINISTRATION 4. COMPUTER SCIENCE
6. POLITICAL SCIENCE 5. MECHINICAL ENGINEERING
7. ECONOMICS OTHER TELEGRAM CHANNELS
8 PHYSICS 1 GOVERNMENT JOBS
9 COMMERCE ACCOUNTANCY 2 LEARN YOGA & MEDITATION
10 ANTHROPOLOGY 3 LEARN ENGLISH
11 LAW 4 BEST DELAS & OFFERS
12 PHILOSOPHY 5 IAS HINDI BOOKS
13 CHARTERED ACCOUNTANTANCY 6 PDFs FOR ALL EXAMS
14 MEDICAL SCIENCE 7. WORLD DIGITAL LIBIRARY
1.CHENNAI STUDENTS 2.BANGLORE STUDENTS 3. CURRENT AFFAIRS
CONTACT FOR ADVERTISEMENT IN ABOVE CHANNLES
ADMIN1: ADMIN2:
SHANKAR TM

IAS ACADEMY
4 The Best IAS Academy In South India SINCE 2004

55. The Happy Seeder Technology is mainly used in India for which purposes?
(a) To ease the process of seeding
( b) An alternative to the stubble burning
(c) A solution to the GM crops
(d ) A kind of Rain water harvesting technology

EXPLANATION:

Happy Seeder , the tractor - mounted device has the capability to cut and lift the previous
crop (in this case the rice straw) and sow a new (wheat ) crop in its place . So , Option ( b )
is the correct .
ADDITIONAL INFORMATION:

Air pollution is a worry especially in north India. Stubble burning is said to be a key factor
behind the formation of a dense cover of smog in this part of India though its contribution
is less than 20%. The stubble is burnt by farmers ahead of the sowing season . They say
they have no option but to set fire to the remnants of the previous harvest before they can
start the seeding process afresh , even though it results in copious amount of smoke being
emitted into the atmosphere . But a new technology that was tested last year holds out
hope , Called Happy Seeder , the tractor- mounted device has the capability to cut and lift the
previous crop (in this case the rice straw) and sow a new (wheat) crop in its place . It also
deposits the straw over the sown area as mulch .

SOURCE:

https: / / www.thehindu .com / news / national / other- states / punjabs- burning-


problem / article25339426 . ece

56. Recently , the Canine Distemper Virus ( CDV ) infection was found at which of
the following protected areas in India?
(a) Bandhavgarh National Park
( b) Kasiranga Wildlife Sanctuary
(c) Gir Wildlife Sanctuary
(d ) Periyar Wildlife Sanctuary

EXPLANATION:

A canine distemper virus ( CDV ) has led to many deaths in the Gir sanctuary. In
September alone , 21 lions died . Every year , it is normal for 80-90 lions to die in Gujarat ,
according to wildlife experts . So, option (c ) is correct .

ADDITIONAL INFORMATION:
57
SHANKAR TM

IAS ACADEMY
^ The Best IAS Academy In South India SINCE 2004

What is Canine Distemper?

This is a viral disease that is frequent in dogs , foxes , wolves , big cats and even primates . It
is caused by a single-stranded RNA virus of the family Paramyxoviridae (the family of
viruses causing measles , mumps and bronchiolitis in humans) . It infects the spinal cord
and brain and also the respiratory and gastrointestinal tracts . The virus is believed to have
a 50% fatality rate in dogs .

In 1994 , the CDV was responsible for an epidemic in the Serengeti region of Africa, where
1 , 000 lions died in three weeks . Its prevalence in India has not been studied and only a few
reports are available regarding its detection in wild carnivores.

SOURCE:

https: / / www.thehindu . com / opinion / op-ed / containing- a- virus / article25183242 .ece

57. Consider the following statements about the Jammu and Kashmir
Reorganisation Act , 2019:

1 . It provides for reorganisation of the state of Jammu and Kashmir into the
Union Territory of Jammu and Kashmir and Union Territory of Ladakh .
2. It provides for a Legislative Assembly with 5 years tenure and Legislative
Council with 6 years tenure for the Union Territory of Jammu and Kashmir .
3. The High Court of Jammu and Kashmir will be the common High Court for the
Union Territories of Ladakh and Jammu and Kashmir.
4. The Union Territory of Jammu and Kashmir will be administered by the
President , through an administrator appointed by him known as the Lieutenant
Governor.
Which of the statements given above are correct?

(a) 1 , 2 and 3 only


( b) 1 , 2 and 4 only
(c) 1 , 3 and 4 only
(d ) 2 , 3 and 4 only

EXPLANATION:

The Bill reorganises the state of Jammu and Kashmir into: (i) the Union Territory of
Jammu and Kashmir with a legislature , and (ii) the Union Territory of Ladakh without a
legislature . The Union Territory of Ladakh will comprise Kargil and Leh districts, and the
Union Territory of Jammu and Kashmir will comprise the remaining territories of the
existing state of Jammu and Kashmir . So , Statement 1 is correct .

58
SHANKAR TM

IAS ACADEMY
4 The Best IAS Academy In South India SINCE 2004

The Bill provides for a Legislative Assembly for the Union Territory of Jammu and
Kashmir. The total number of seats in the Assembly will be 107 . Of these , 24 seats will
remain vacant on account of certain areas of Jammu and Kashmir being under the
occupation of Pakistan . The Assembly will have a term of five years, and the Lieutenant
Governor must summon the Assembly at least once in six months . No there is no
provision for Legislative Council. So , statement 2 is not correct .
The High Court of Jammu and Kashmir will be the common High Court for the Union
Territories of Ladakh , and Jammu and Kashmir . Further , the Union Territory of Jammu
and Kashmir will have an Advocate General to provide legal advice to the government of
the Union Territory. So , Statement 3 is correct .
The Union Territory of Jammu and Kashmir will be administered by the President ,
through an administrator appointed by him known as the Lieutenant Governor . The
Union Territory of Ladakh will be administered by the President , through a Lieutenant
Governor appointed by him . So , Statement 4 is correct .

ADDITIONAL INFORMATION:

 The Jammu and Kashmir Reorganisation Bill , 2019 was introduced in Rajya Sabha
on August 5 , 2019 by the Minister of Home Affairs , Mr . Amit Shah . The Bill provides
for reorganisation of the state of Jammu and Kashmir into the Union Territory of
Jammu and Kashmir and Union Territory of Ladakh .

 Reorganisation of Jammu and Kashmir : The Bill reorganises the state of Jammu and
Kashmir into: (i) the Union Territory of Jammu and Kashmir with a legislature , and
(ii) the Union Territory of Ladakh without a legislature . The Union Territory of
Ladakh will comprise Kargil and Leh districts, and the Union Territory of Jammu
and Kashmir will comprise the remaining territories of the existing state of Jammu
and Kashmir .

 Lieutenant Governor: The Union Territory of Jammu and Kashmir will be


administered by the President , through an administrator appointed by him known
as the Lieutenant Governor . The Union Territory of Ladakh will be administered by
the President , through a Lieutenant Governor appointed by him .

 Legislative Assembly of Jammu and Kashmir : The Bill provides for a Legislative
Assembly for the Union Territory of Jammu and Kashmir . The total number of seats
in the Assembly will be 107. Of these , 24 seats will remain vacant on account of
certain areas of Jammu and Kashmir being under the occupation of
Pakistan . Further , seats will be reserved in the Assembly for Scheduled Castes and

59
SHANKAR TM

IAS ACADEMY
^ The Best IAS Academy In South India SINCE 2004

Scheduled Tribes in proportion to their population in the Union Territory of Jammu


and Kashmir. In addition , the Lieutenant Governor may nominate two members to
the Legislative Assembly to give representation to women , if they are not adequately
represented .

 The Assembly will have a term of five years, and the Lieutenant Governor must
summon the Assembly at least once in six months. The Legislative Assembly may
make laws for any part of the Union Territory of Jammu and Kashmir related to: (i)
any matters specified in the State List of the Constitution , except “Police” and
“ Public Order” , and (ii) any matter in the Concurrent List applicable to Union
Territories. Further , Parliament will have the power to make laws in relation to any
matter for the Union Territory of Jammu and Kashmir.

 Council of Ministers: The Union Territory of Jammu and Kashmir will have a Council
of Ministers of not more than ten percent of the total number of members in the
Assembly. The Council will aide and advise the Lieutenant Governor on matters that
the Assembly has powers to make laws. The Chief Minister will communicate all
decisions of the Council to the Lieutenant Governor.

 High Court: The High Court of Jammu and Kashmir will be the common High Court
for the Union Territories of Ladakh , and Jammu and Kashmir. Further , the Union
Territory of Jammu and Kashmir will have an Advocate General to provide legal
advice to the government of the Union Territory.

 Legislative Council: The Legislative Council of the state of Jammu and Kashmir will
be abolished . Upon dissolution , all Bills pending in the Council will lapse .

 Advisory Committees: The central government will appoint Advisory Committees , for
various purposes, including: (i) distribution of assets and liabilities of corporations of
the state of Jammu and Kashmir between the two Union Territories , (ii) issues
related to the generation and supply of electricity and water , and (iii) issues related
to the Jammu and Kashmir State Financial Corporation . These Committees must
submit their reports within six months to the Lieutenant Governor of Jammu and
Kashmir , who must act on these recommendations within 30 days.

 Extent of laws: The Schedule lists 106 central laws that will be made applicable to
Union Territories of Jammu and Kashmir and Ladakh on a date notified by the
central government . These include the Aadhaar Act , 2016 , the Indian Penal Code ,
1860 , and the Right to Education Act , 2009 . Further , it repeals 153 state laws of
Jammu and Kashmir. In addition , 166 state laws will remain in force , and seven
60
SHANKAR TM

IAS ACADEMY
4 The Best IAS Academy In South India SINCE 2004

laws will be applicable with amendments. These amendments include lifting of


prohibitions on lease of land to persons who are not permanent residents of Jammu
and Kashmir.

SOURCE:

https: / / www.prsindia.org / billtrack / jammu -and- kashmir- reorganisation - bill- 2019

58 . Which among the following river valleys under the influence of intensive gully
erosion?
(a) Damodar
( b) Kosi
(c ) Brahmaputra

( d ) Chambal
EXPLANATION:

Gully erosion is common on steep slopes . Gullies deepen with rainfall , cut the agricultural
lands into small fragments and make them unfit for cultivation . A region with a large
number of deep gullies or ravines is called a badland topography. Ravines are widespread ,
in the Chambal basin .

SOURCE:

http: / / ncert . nic.in / textbook / pdf / kegyl 06 .pdf

59 . Which among the following is the only industry in India which is self-reliant
and complete in the value chain from raw material to highest value added
products?
(a) Iron and steel industry
(b ) Textile industry
(c ) Fertilizer industry
( d ) Cement industry
EXPLANATION:

Textile Industry

It occupies unique position in the Indian economy, because it contributes significantly to


industrial production ( 14 per cent ) , employment generation (35 million persons directly -

61
SHANKAR TM

IAS ACADEMY
^ The Best IAS Academy In South India SINCE 2004

the second largest after agriculture ) and foreign exchange earnings (about 24.6 per cent ) . It
contributes 4 per cent towards GDP. It is the only industry in the country , which is self-
reliant and complete in the value chain i.e . , from raw material to the highest value added
products .

60 . With reference to the major ports in India , consider the following statements:
1 . Marmagao port was the first port developed soon after Independence .
2. Kandla port accounts for about fifty percent of India’s Iron ore export .
Which of the statements given above is / are correct?

(a) 1 only
( b) 2 only
(c ) Both 1 and 2
( d ) Neither 1 nor 2
EXPLANATION:

Kandla in Kuchchh was the first port developed soon after Independence to ease the
volume of trade on the Mumbai port , in the wake of loss of Karachi port to
Pakistan after the Partition . So , Statement 1 is not correct .

Marmagao port ( Goa) is the premier iron ore exporting port of the country. This port
accounts for about fifty per cent of India ' s iron ore export . So , Statement 2 is not
correct .

61 . Consider the following statements:


1 . Only four Indian States shares land boundary with neighbouring countries of
India .
2. India shares longest land boundary with China than other countries .
3. Indus , Ganga and Brahmaputra are Transboundary Rivers and they flow over
three countries each .
Which of the statements given above is / are not correct?

(a) 2 and 3 only


( b) 1 and 3 only
(c ) 1 , 2 and 3
( d ) 1 only

62
SHANKAR TM

IAS ACADEMY
^ The Best IAS Academy In South India SINCE 2004

EXPLANATION:

There are 10 states which share their International border with neighbouring countries .
These States from east to west are Nagaland , Manipur , Tripura, Meghalaya, Bihar ,
Uttar Pradesh , Himachal Pradesh , Punjab , Rajasthan and Gujarat . So , Statement 1 is
not correct .

India shares longest land boundary with Bangladesh . So , Statement 2 is not correct .
The Ganges River flows through parts of the territories of the two countries of India and
Bangladesh , and along the way covers a distance of 2 , 520 kilometers ( 1 , 560 miles) .
Indus and Brahmaputra flows through 3 countries . So , Statement 3 is not correct .

62. Which among the following States has maximum proportion of elderly people in
its population?
(a) Assam
( b) Haryana
(c ) Kerala
( d ) Uttarakhand
EXPLANATION:

Who is called a Senior citizen?

According to the law , a " senior citizen " means any person being a citizen of India,
who has attained the age of sixty years or above .

Status in India:

 According to Population Census 2011 there are nearly 104 million elderly
persons (aged 60 years or above ) in India ; 53 million females and 51 million
males . A report released by the United Nations Population Fund and HelpAge
India suggests that the number of elderly persons is expected to grow to 173
million by 2026 .
 Both the share and size of elderly population is increasing over time . From 5.6%
in 1961 the proportion has increased to 8.6% in 20 11 . For males it was
marginally lower at 8.2% , while for females it was 9.0%.
 As regards rural and urban areas, 71% of elderly population resides in rural
areas while 29 % is in urban areas.
 The life expectancy at birth during 2009- 13 was 69.3 for females as against 65.8
years for males . At the age of 60 years average remaining length of life was found
to be about 18 years ( 16.9 for males and 19.0 for females) and that at age 70 was

63
SHANKAR TM

IAS ACADEMY
4 The Best IAS Academy In South India SINCE 2004

less than 12 years ( 10.9 for males and 12.3 for females ) . Kerala has got the
highest life expectancy at birth , followed by Maharashtra and Punjab. The life
expectancy at birth in Kerala is 71.8 years and 77.8 years for males and females
respectively as per the SRS Report 2009 - 13.
 For 2013, the age specific death rate per 1000 population for the age group 60 -
64 years was 19.7 for rural areas and 15.0 for urban areas . Altogether it was
18.4 for the age group 60 - 64 years. As regards , sex - wise , it was 20.7 for males
and 16.1 for females .
 The old - age dependency ratio climbed from 10.9% in 1961 to 14.2% in 2011 for
India as a whole . For females and males , the value of the ratio was 14.9 % and
13.6% in 2011.
 In rural areas, 66% of elderly men and 28% of elderly women were working, while
in urban areas only 46% of elderly men and about 11% of elderly women were
working.
 The percent of literates among elderly persons increased from 27% in 1991 to
44 % in 2011.The literacy rates among elderly females ( 28% ) is less than half of
the literacy rate among elderly males (59% ) .
 Prevalence of heart diseases among elderly population was much higher in urban
areas than in rural parts .
 Most common disability among the aged persons was locomotor disability and
visual disability as per Census 2011.
 In the age - group of 60 - 64 years, 76% persons were married while 22% were
widowed . Remaining 2% were either never married or divorced .
 State - wise data on elderly population divulge that Kerala has maximum
proportion of elderly people in its population ( 12.6 per cent ) followed by Goa ( 11.2
per cent ) and Tamil Nadu ( 10.4 per cent ) as per Population Census 2011 . he least
proportion is in Dadra & Nagar Haveli ( 4.0 per cent ) followed by Arunachal
Pradesh ( 4.6 per cent ) and Daman & Diu and Meghalaya ( both 4.7 per cent ).

63. Consider the following pairs:

Lake Origin

1. Lonar lake Tectonic


2. Gangabal lake Glacial
3. Purbasthali lake Fluvial

4. Bhimtal lake Crater

64
SHANKAR TM

IAS ACADEMY
4 The Best IAS Academy In South India SINCE 2004

Which of the Pairs given above is / are correctly matched?

(a) 1 only
( b) 1 and 3 only
(c ) 2 and 3 only
(d ) 1, 2 , 3 a n d 4

EXPLANATION:

Lonar lake is a lake of lagoon in Buldhana district of Maharashtra state . It was created
due to a meteorite . So , Pair 1 is not correctly matched .

Gangabal lake is fed by precipitation , glaciers and springs . So , Pair 2 is correctly


matched.

Purbasthali is a large block with a number of villages adjoining a large oxbow lake ,
created by the river Ganga, on its Western bank , in PurbaBardhaman district of West
Bengal. It is 120 km north from Calcutta. So, Pair 3 is correctly matched .

Bhimtal Lake lunar-shaped natural freshwater lake was born from a depression in the
Nainital town of the Kumaon region , owing to tectonic movement. So , Pair 4 is not
correctly matched .

64. Consider the following statements about the Okjokull glacier:


1. It is the first to lose its status as a glacier due to the climate change .
2. It is located in Greenland .

Which of the statements given above is / are correct?

(a) 1 only
( b) 2 only
(c) Both 1 and 2
(d ) Neither 1 nor 2

EXPLANATION:

Iceland recently honoured the passing of Okjokull, its first glacier lost to climate change .
So , Statement 1 is correct and Statement 2 is not correct.

65
SHANKAR TM

IAS ACADEMY
^ The Best IAS Academy In South India SINCE 2004

ADDITIONAL INFORMATION:

A bronze plaque was unveiled in a ceremony starting around 1400 GMT to mark Okjokull --

which translates to “Ok glacier ” -- in the west of Iceland . The plaque bears the inscription
“ A letter to the future ,” and is intended to raise awareness about the decline of glaciers and
the effects of climate change .Iceland loses about 11 billion tonnes of ice per year , and
scientists fear all of the island country’s 400- plus glaciers will be gone by 2200 , according
to Howe and her Rice University colleague Dominic Boyer . Glaciologists stripped Okjokull of
its glacier status in 2014 , a first for Iceland .

SOURCE:

https: / / www. thehindubusinessline . com / news / science / iceland - commemorates-first-


glacier-lost- to-climate -change / article29125454.ece #

65 . Consider the following statements about the One Nation and One Ration:
1. The key focus of this scheme is ensuring that beneficiaries covered under
National Food Security Act are not denied their rations because of inter-state
migration .
2. Currently all the states in India have joined in this scheme .
3. Under this , only the inter-state portability of ration card , not intra -state
portability , will be implemented .
Which of the statements given above is / are correct?

(a) 1 only
( b) 1 and 2 only
(c) 2 and 3 only
(d ) 1 , 2 and 3

EXPLANATION:

The main objective of the scheme is to introduce nation - wide portability of ration card
holders under National Food Security Act , 2013 ( NFSA) , to lift their entitlement foodgrains
from any FPS in the country without the need to obtain a new ration card , by integrating
the existing PDS systems / portals of States / UTs with the Central systems / portals, etc . So ,
Statement 1 is correct .
At present , 9 states have agreed to start the scheme and have identified the districts . These
states are Andhra Pradesh , Gujarat , Karnataka, Kerala , Maharashtra, Uttar Pradesh ,
Orissa, Tamil Nadu and Assam . The remaining states have been requested to select the
districts and work on implementation of this system by selecting Agency- Mills for rice

66
SHANKAR TM

IAS ACADEMY
4 The Best IAS Academy In South India SINCE 2004

fortification . So, Statement 2 is not correct .


both intra-state and inter-state portability of ration card is being implemented under this
scheme . So, Statement 3 is not correct .

SOURCE:

https: / / www.livemint .com / news / india / -one- nation-one-ration -card-inter-state- portability-


across-india- by- june- 2020- 1565360235091. html

66 . With respect to the United Nations Convention to Combat Desertification ,


which among the following statements is not correct?
(a) It is the sole legally binding international agreement linking environment and
development to sustainable land management
( b) India hosted the Conference of Parties of this convention for the first time recently
(c) It was an outcome the United Nations Conference on Sustainable Development
in 2012 or simply known as Rio + 20 summit
(d ) The Convention addresses specifically the arid , semi-arid and dry sub-humid areas ,
known as the drylands , where some of the most vulnerable ecosystems and peoples
can be found
EXPLANATION:

UNCCD was not an outcome the United Nations Conference on Sustainable Development
in 2012 , but it was the outcome of Rio de Janeiro conference in 1992 . So option ( c ) is
not correct .

ADDITIONAL INFORMATION:

 In 1977 the United Nations Conference on Desertification ( UNCOD ) adopted a Plan of


Action to Combat Desertification ( PACD ) . Despite this and other efforts , the United
Nations Environment Programme ( UNEP) concluded in 1991 that the problem of
land degradation in arid , semi-arid and dry sub-humid areas had intensified ,
although there were “local examples of success”. As a result , the question of how to
tackle desertification was still a major concern for the United Nations Conference on
Environment and Development ( UNCED ) , which was held in Rio de Janeiro in 1992 .
The Conference supported a new, integrated approach to the problem , emphasizing
action to promote sustainable development at the community level.
 The Rio Conference called on the United Nations General Assembly to establish an
Intergovernmental Negotiating Committee ( INCD ) to prepare , by June 1994 , a
Convention to Combat Desertification , particularly in Africa. In December 1992 , the

67
SHANKAR TM

IAS ACADEMY
4 The Best IAS Academy In South India SINCE 2004

General Assembly agreed and adopted resolution 47 / 188 on this matter. Working to
a tight schedule , the Committee completed its negotiations in five sessions .
 The Convention was adopted in Paris on 17 June 1994 and entered into force on 26
December 1996 , 90 days after the 50 th ratification was received . 196 countries and
the European Union are Parties as at August 2018 . The Conference of the Parties
( COP) , which is the Convention ' s supreme governing body , held its first session in
October 1997 in Rome , Italy.

SOURCE:

https: / / www.unccd .int / convention / about-convention / unccd - history

67. Which of the following is / are the objective (s ) of the Jal Jeevan Mission?
1. It aims to provide piped water for every households across the country .
2. It focuses on groundwater recharge , rainwater harvesting and management of
household wastewater for reuse in agriculture .
3. It strives to achieve linking the surplus river water regions to deficit river
water regions by linking the rivers across the country .
Select the correct answer using the code given below .

(a) 1 only
( b) 1 and 2 only
(c) 2 and 3 only
(d ) 1 , 2 and 3

EXPLANATION:

The objective of linking the surplus river water regions to deficit river water regions by
linking the
rivers across the country is under the Interlinking of River ( ILR ) programme . So ,
Statement 3 is not correct .

ADDITIONAL INFORMATION:

Jal Jeevan Mission , a central government initiative under the Ministry of Jal Shakti , aims
to ensure access of piped water for every household in India.Jal Jeevan Mission will be a
decentralised , community- managed and sustainable water management scheme . This
programme forms a crucial part of the Jal Jeevan Mission . The programme aims to
implement source sustainability measures as mandatory elements, such as recharge and
reuse through grey water management , water conservation , and rain water harvesting . The
Jal Jeevan Mission will be based on a community approach to water . According to the
68
SHANKAR TM

IAS ACADEMY
4 The Best IAS Academy In South India SINCE 2004

government , the mission will include information , education and communication as key
components. The mission is meant to create a people ' s movement for water , making it
everyone’s priority.

SOURCE:

https: / / www.business- standard . com / about / what-is- jal- jeevan - mission

68 . Which of the following Cities is / are located on river bank?


1 . Bhagalpur

2. Trichy
3. Brahmapur
4. Kozhikode

Select the correct answer using the code given below .

(a) 1 and 3 only


( b) 2 and 4 only
(c ) 1 , 2 and 3 only

( d ) 1 , 2, 3 and 4
EXPLANATION:

Bhagalpur is located on the bank of River Ganga .


Trichy is located on the bank of River Cauvery ( Kaveri) .
Brahmapur is located on the bank of River Rushikulya.
Kozhikode is located on the bank of River Chaliyar . So , Option ( d ) is correct .

69 . Consider the following pairs:

Hot springs Located in

1 Rishikund Uttarakhand

2 Chumathang Jammu and Kashmir

3 Dhuni Pani Madhya Pradesh

4 Reshi Sikkim

Which of the pairs given above is / are correctly matched ?


69
SHANKAR TM

IAS ACADEMY
^ The Best IAS Academy In South India SINCE 2004

(a) 1 only
( b) 2 and 3 only
(c ) 1 , 2 and 4 only
( d ) 1 , 2, 3 and 4

EXPLANATION:

Rishikund is a holy pond located near Raghunath temple in Rishikesh , Uttarakhand . So ,


Pair 1 is correctly matched .

Chumathang is a village in Leh District , Jammu and Kashmir . So , Pair 2 is correctly


matched.

Dhuni Pani is a natural hot spring 8s famous tourist destinations in Amarkantak , Madhya
Pradesh . So , Pair 3 is correctly matched .

Reshi hot spring on the banks of Rangeet River in Sikkim . So , Pair 4 is correctly
matched.

70 . With reference to Hydropower inlndia , consider the following statements:


1. Hydro projects with installed capacity less than 35 Megawatts are considered as
small hydro projects .
2 . While Ministry of Power is responsible for large hydro projects, the small hydro
power is given to Ministry of New and Renewable Energy .
Which of the statements given above is / are correct?

(a) 1 only
(b ) 2 only
(c ) Both 1 and 2
( d ) Neither 1 nor 2
EXPLANATION:

In India , hydro projects up to 25 MW station capacities have been categorized as Small


Hydro Power (SHP) projects . So , Statement 1 is not correct .

While Ministry of Power , Government of India is responsible for large hydro projects , the
mandate for the subject small hydro power (up to 25 MW ) is given to Ministry of New
and Renewable Energy. So , Statement 2 is correct .

70
SHANKAR TM

IAS ACADEMY
^ The Best IAS Academy In South India SINCE 2004

ADDITIONAL INFORMATION:

Hydro power projects are generally categorized in two segments i.e . small and large hydro .
In India, hydro projects up to 25 MW station capacities have been categorized as Small
Hydro Power (SHP) projects .

 Micro: upto 100 KW


 Mini: l O l K W t o 2 MW
 Small: 2 MW to 25 MW
 Mega: Hydro projects with installed capacity > = 500 MW
 Thermal Projects with installed capacity > = 1500 MW

71 . Arrange the following from West to Bast:

1 . Mizo hills
2. Patkai bum
3. Khasi hills
4. Jaintia hills
Select the correct answer using the code given below .

( a ) 3 -4 - 1 -2
( b) 3- 4- 2 - 1
(c ) 2 -3- 4 - 1
( d ) 2 - 1 - 4 -3
EXPLANATION:

Tsangpo
GREAT HIMALAYAS
*&
ft ri

BENGAL l

So , Option (a ) is correct .

71
SHANKAR TM

IAS ACADEMY
4 The Best IAS Academy In South India SINCE 2004

72. Consider the following statements about Rare Barth Minerals:


1. These are very rare in quantity and some of them are very scarce in earth’s
crust .
2. They are also referred to as "rare earth oxides" because many of them are
typically sold as oxide compounds .
3. China is the world’s largest producer and consumer of rare earth minerals .

Which of the statements given above are correct?

(a) 1 and 2 only


( b) 1 and 3 only
(c) 2 and 3 only
(d ) 1 , 2 and 3

EXPLANATION:

Despite their name , rare -earth elements are - with the exception of the radioactive
promethium - relatively plentiful in Earth ' s crust , with cerium being the 25th most
abundant element at 68 parts per million , more abundant than copper .So , Statement 1
is not correct .

They are also referred to as " rare earth oxides" because many of them are typically sold
as oxide compounds . So , Statement 2 is correct .

In addition to being the world ' s largest producer of rare earth materials, China is also the
dominant consumer . They use rare earths mainly in manufacturing electronics products
for domestic and export markets .So , Statement 3 is correct .

73. Consider the following statements about Hyperloop Transmission System:


1 . It travels faster than Maglev trains but slower than airplanes .
2. The Pod is designed to make air move faster along the top and thereby
decreasing air pressure to overcome air resistance during motion .
3. To stop motion , brakes are applied by the use of electromagnetic force .

Which of the statements given above is / are correct?

(a) 1 only
( b) 2 and 3 only
(c ) 1 , 2 and 3
( d ) None of the above

72
SHANKAR TM

IAS ACADEMY
^ The Best IAS Academy In South India SINCE 2004

EXPLANATION:

Hyperloops travel faster than maglev trains and travels nearly equal to or even faster than
airplanes . So , Statement 1 is not correct .
Pods travel in near vacuum chamber where air is removed to reduce friction . So ,
Statement 2 is not correct .
Electromagnetic forces are used in wheels of trains . But Hyperloops ( does not have wheels )
use magnetic levitation . So , Statement 3 is not correct .

ADDITIONAL INFORMATION:

What is Hyperloop?
Hyperloop is a new form of ground transport currently in development by a number of companies, It
could see passengers travelling at over 700 miles an hour in floating pod which races along inside
giant low- pressure tubes, either above or below ground .

What makes Hyperloop different?


There are two big differences between Hyperloop and traditional rail. Firstly , the pods carrying
passengers travel through tubes or tunnels from which most of the air has been removed to reduce
friction . This should allow the pods to travel at up to 750 miles per hour.
Secondly , rather than using wheels like a train or car , the pods are designed to float on air skis,
using the same basic idea as an air hockey table , or use magnetic levitation to reduce friction .

What are the benefits of Hyperloop?


Supporters argue that Hyperloop could be cheaper and faster than train or car travel , and cheaper
and less polluting than air travel. They claim that it ' s also quicker and cheaper to build than
traditional high- speed rail. Hyperloop could therefore be used to take the pressure off gridlocked
roads, making travel between cities easier, and potentially unlocking major economic benefits as a
result .

SOURCE:

https: / / www.hyperloop . global /

74. Recently the First India - Caribbean summit was organized in New York . Which
of the following countries has participated in that summit representing the
Caribbean Community organization or simply known as CARICOM organization?
1. Bahamas
2. Barbados
3. Brazil
4. Trinidad and Tobago

Select the correct answer using the code given below .

(a) 1 , 2 and 3 only


( b) 1 , 2 and 4 only
73
SHANKAR TM

IAS ACADEMY
^ The Best IAS Academy In South India SINCE 2004

(c) 1 , 3 and 4 only


(d ) 2 , 3 and 4 only

EXPLANATION:

Leaders from Antigua & Barbuda, Bahamas, Barbados, Belize , Dominica , Grenada ,
Guyana, Haiti, Jamaica, Saint Kitts and Nevis , Saint Lucia, Saint Vincent 8s the
Grenadines, Suriname , Trinidad and Tobago attended the the First India - Caribbean
summit.

SOURCE:

https: / / www.business- standard . com / article / pti- stories / pm- modi- hosts-flrst-ever-india-
caricom -leaders- meeting- 119092600063_ l .html

75. With reference to judiciary in India , consider the following statements:


1. The strength of the Supreme Court Judges is determined by the law of Indian
Parliament .
2. The strength of the High Court Judges is determined by the law of respective
State Legislature .
3. The strength of the High Court Judges across the country is not uniform .

Which of the statements given above are correct?

(a) 1 and 2 only


( b) 1 and 3 only
(c) 2 and 3 only
(d ) 1 , 2 and 3

EXPLANATION:

The Indian constitution under Article 124 ( 1) states that there shall be a Supreme Court
of India constituting of a Chief Justice of India (CJI ) and , until Parliament by law
prescribes a larger number , of not more than seven other Judges . So , Statement 1 is
correct .

Judges in a high court are appointed by the President of India in consultation with
the Chief Justice of India and the governor of the state. High courts are headed by a
chief justice . The chief justices rank fourteenth (within their respective states) and
seventeenth (outside their respective states) on the Indian order of precedence . The
number of judges in a court is decided by dividing the average institution of main cases
during the last five years by the national average , or the average rate of disposal of main
cases per judge per year in that High Court , whichever is higher .So , Statement 2 is not
correct .

74
SHANKAR TM

IAS ACADEMY
^ The Best IAS Academy In South India SINCE 2004

The number of judges in a court is decided by dividing the average institution of main
cases during the last five years by the national average , or the average rate of disposal of
main cases per judge per year in that High Court , whichever is higher . So , Statement 3
is correct .

76 . Why deserts are not located on the eastern side of continents?


1 . Absence of offshore trade winds .
2. Absence of cold ocean current .
Select the correct answer using the code given below .

(a) 1 only
( b) 2 only
(c ) Both 1 and 2
( d ) Neither 1 nor 2
EXPLANATION:

Tropical deserts are located between 10 ° to 30 ° north and south of the equator on the
western margin of the continents, for example , Atacama Desert in South America.
Californian desert in North America , Namib and Kalahari Desert in Southern Africa , Sahara
and West Australian desert.

There are four major factors responsible for the location of the world ' s tropical desert on the
Western margins of continents:

1. Offshore areas of trade wind and falling under the rain shadow area:

When the moisture laden trade wind flows from east to west shed their moisture on the
eastern part and by the time they reach the western margin , they become dry. These dry
winds make the soil more and this led to the formation of the desert .’

2 . Anticyclonic conditions:

The areas between 20 0


- 30 ° latitude on western margins of continents are the regions of
descending air . Because of this, the air gets compressed and warm as it descends and thus
the moisture keeps decreasing .

3. Formation of Rain -shadow Zone :

A region in the lee of mountains that receives less rainfall than the region windward of the
mountains is called rain -shadow zone .

75
SHANKAR
HI <
IAS ACADEMY
The Best IAS Academy In South India SINCE 2004
TM

Rain- shadow Zone

A Dry air

Ocean or Sea

In the above image , you saw how mountain is acting as a barrier for prevents orographic
rainfall . For example : Thar Desert in India is formed due to the formation of rain - shadow
zone because Aravallis mountains are situated parallel to the region . Therefore the
moisture holding winds pass away from the region because there is absence of mountain
barriers.

4 . Presence of cold ocean currents along the western coast of continents tends to stabilise
the air over the coast . This prevents cloud formation and rainfall . Hence , it leads to arid
conditions or the formation of marine deserts on the adjacent coastal lands .

77. Consider the following statements about Tropical dry evergreen forest:
1 . It is a unique forest type found only in narrow coastal strip of Tamilnadu and
Andhra Pradesh.
2. It has dissymmetric climatic condition where rainfall occurs both during
summer and winter monsoon .
3. Vedanthangal Bird Sanctuary lies in tropical dry evergreen forest of Tamilnadu.
Select the correct answer using the code given below .

(a ) 1 and 2 only
( b) 2 and 3 only
(c ) 1 and 3 only
( d ) 1 , 2 and 3

76
SHANKAR TM

IAS ACADEMY
^ The Best IAS Academy In South India SINCE 2004

EXPLANATION:

The Tropical Dry Evergreen Forest is a unique forest found only in Tamil Nadu and
Andhra Pradesh . It is distribution is restricted to the narrow coastal strip from
Vishakapattinam in Andhra Pradesh in the north to Ramanathapuram in Tamil Nadu in
the south . So , Statement 1 is correct .

The dissymmetric climatic condition of this region , where rainfall occurs both during
summer and winter monsoon and extended the dry season from March to September ,
favors development and sustenance of the Tropical Dry Evergreen forest in this narrow
strip. So , Statement 2 is correct .

Vedanthangal forest type: Tropical wet evergreen forests changes to tropical dry
deciduous forests in the foothills and to thorn forests further down . So , Statement 3 is
not correct .

ADDITIONAL INFORMATION:

The coastal area in Vedaranyam is rich in biodiversity , with many unique species of
animals and birds apart from religious , historical and cultural diversity . Many ecologically
sensitive areas such as mangroves , lagoons , tidal mudflats, swamps and tropical dry
evergreen forests are present . The Point Calimere Wildlife Sanctuary near Vedaranyam is a
unique mix of grasslands , mudflats , backwaters , sand dunes and Tropical Dry Evergreen
Forest (TDEF) . The Tropical Dry Evergreen Forest is a unique forest found only in Tamil
Nadu and Andhra Pradesh . It is distribution is restricted to the narrow coastal strip from
Vishakapattinam in Andhra Pradesh in the north to Ramanathapuram in Tamil Nadu in
the south . The dissymmetric climatic condition of this region , where rainfall occurs both
during summer and winter monsoon and extended the dry season from March to
September , favors development and sustenance of the Tropical Dry Evergreen forest in this
narrow strip . The TDEF has a mixture of trees, shrubs , lianas , and herbs and form a
complete canopy in pristine condition and provide habitat to a wide variety of animals
including insects, amphibians, reptiles , and mammals. However there is hardly any of this
forest in pristine condition and most of them are little more than degraded thorny thickets ,
lacking the inherent nobility of the climax vegetation . The TDEF is currently preserved in
the Point Claimer Wildlife Sanctuary and in the coastal sacred groves . In the ICZM project
in Vedaranyam , an attempt has been made to establish the TDEF in about 66 acres of
village common land with the active participation of Panchayat Raj Institutions and the
local community.

77
SHANKAR TM

IAS ACADEMY
^ The Best IAS Academy In South India SINCE 2004

SOURCE:

http: / / sicom . nic.in / milestones / conservation -and - protection- coastal-


resources / conservation - tropical-dry-evergreen -forests

78 . Which among the following is not correct about the climatic condition of the
South of India?
(a) Less diurnal range of temperature
(b ) The high annual range of temperature
(c ) Temperature is high throughout the year
( d ) Extreme climatic conditions cannot be found

EXPLANATION:

South of India is located in the tropical region . For Example , even during winter , Kerala
receive almost 12 hours of sunlight . So , annual temperature range will not be high . So ,
option (b ) is correct .

79 . Consider the following statements:


1 . The Aravalli range in India spread across three Indian States only .
2. Mangroves in India extend over nine Indian States and three Union Territories
only .
Which of the statements given above is / are correct?

(a) 1 only
( b) 2 only
(c ) Both 1 and 2
( d ) Neither 1 nor 2
EXPLANATION:

The Aravalli Range ( Devanagari, also spelled Aravali range, is a range of mountains
running approximately 692 km ( 430 mi ) in a southwest direction , starting in
North India from Delhi and passing through southern Haryana , through
to Western India across the states of Rajasthan and ending in Gujarat. So , Statement 1
is correct .

Mangroves in India extends over nine states and three union territories only. So ,
Statement 2 is correct .

78
SHANKAR TM

IAS ACADEMY
^ The Best IAS Academy In South India SINCE 2004

ADDITIONAL INFORMATION:

Mangrove cover in Indian coastal states and union territories ( 1987- 2013 )

Assessment Year ( Area in km 2)


State/UT
1987 1989 1991 1993 1995 1997 1999 2001 2003 2005 2009 2011 2013

A&N
Islands
686 973 971 966 966 966 966 789 658 635 615 617 6! I
Andhra
495 405 399 378 383 383 397 333 329 354 353 352 352
Pradesh

Daman &
Diu
0 0 0 0 0 0 0 0 1 1 1 2 ,3

Goa 0 3 3 3 3 5 5 5 16 16 17 22 :2

Gujarat 427 412 397 419 689 901 1.031 911 916 991 1.046 1.058 1 03

Karnataka o 0 0 0 2 3 3 2 3 3 3 3

Kerala 0 0 0 0 0 0 0 0 8 5 5 6

Maharashtra 140 114 113 155 155 124 108 118 158 186 186 186 1

Orissa 199 192 195 195 195 211 215 219 203 217 221 222

Puducherry o 0 0 0 0 0 0 1 1 1 1 1

Tamil Nadu 23 47 47 21 21 21 21 23 35 36 39 39

West
2.076 2, 1'09 2.119 2.119 2.119 2.123 2.125 2.081 2,120 2,136 2.152 2.155 2 097
Bengal*

Total 4,046 4,255 4 , 244 4,256 4,533 4,737 4,871 4,482 4, 448 4,581 4,639 4,663 4.628

SOURCE:

http: / / www frienvis nic in / Database / Mangrove -cover-in -India_1827 aspx


. . . .

80. Consider the following statements about El- Nino southern oscillation ( ENSO ):
.
1 The phenomenon of periodic development of a warm ocean current along the
coast of Peru as a temporary replacement of the cold Peruvian current every 2
to 5 years is called El- Nino .
.
2 El Nino Southern oscillation is a condition where tropical South -eastern Pacific
Ocean experiences high pressure and tropical eastern Indian Ocean experiences
low pressure .
Select the correct answer using the code given below .
(a ) 1 only
( b) 2 only
(c ) Both 1 and 2
(d ) Neither 1 nor 2

79
SHANKAR TM

IAS ACADEMY
^ The Best IAS Academy In South India SINCE 2004

EXPLANATION:

The phenomenon of periodic development of a warm ocean current along the coast of
Peru as a temporary replacement of the cold Peruvian current every 2 to 5 years is called
El- Nino . So , Statement 1 is correct .

Southern oscillation is a condition where tropical eastern south Pacific Ocean


experiences Low pressure and tropical eastern Indian Ocean experiences high pressure .
So , Statement 2 is not correct .

SOURCE:

http: / / ncert . nic.in / ncerts /1/ iess104 . pdf

81 . India is planning to create the capacity of producing 175 Gigawatt energy from
renewable energy sources. In this context , arrange the following sources of energy
in the order from high to low as per the target .

1 . Solar Energy
2. Wind Energy
3. Small hydro Energy
4. Biomass Energy

Select the correct answer using the code given below .

(a) 1 - 2 - 3 - 4
( b) 2 - 1 - 3 - 4
(c ) 1 - 2 - 4 - 3
(d ) 2 - 1 - 4 - 3
EXPLANATION:

A target of installing 175 GW of renewable energy capacity by the year 2022 has been
set , which includes 100 GW from solar , 60 GW from wind , 10 GW from bio- power and 5
GW from small hydro- power .

82. Recently the Sample Registration System ( SRS ) under the Office of The
Registrar General of Indiameasured that Maternal Mortality Rate ( MMR ) in India
has declined . Which of the following initiatives helped to achieve that decline in
India?
1. Janani Suraksha Yojana
2. Janani Shishu Suraksha Karyakram
80
SHANKAR TM

IAS ACADEMY
^ The Best IAS Academy In South India SINCE 2004

3. Pradhan MantriMatruVandanaYojana
4. PoshanAbhiyan

Select the correct answer using the code given below .

(a) 1 and 2 only


( b) 1 and 3 only
(c) 2 and 3 only
(d ) 1 , 2 , 3 and 4

EXPLANATION:

Focus on quality and coverage of health services through public health initiatives under
the National Health Mission such as LaQshya , PoshanAbhiyan , Pradhan
MantriSurakshitMatritvaAbhiyan , Janani Shishu Suraksha Karyakram , Janani
Suraksha Yojana and Pradhan MantriMatruVandanaYojana have contributed to the
decline in MMR in India. So , Option ( d ) is correct .

ADDITIONAL INFORMATION:

 Janani Suraksha Yojana (JSY) is a safe motherhood intervention under the National
Rural Health Mission ( NHM ) . It is being implemented with the objective of reducing
maternal and infant mortality by promoting institutional delivery among pregnant
women . The scheme is under implementation in all states and Union Territories
( UTs ) , with a special focus on Low Performing States ( LPS ) . The scheme focuses on
poor pregnant woman with a special dispensation for states that have low
institutional delivery rates, namely, the states of Uttar Pradesh , Uttarakhand , Bihar ,
Jharkhand , Madhya Pradesh , Chhattisgarh , Assam , Rajasthan , Orissa, and Jammu
and Kashmir . While these states have been named Low Performing States ( LPS) , the
remaining states have been named High Performing states ( HPS ) . The scheme also
provides performance based incentives to women health volunteers known as ASHA
(Accredited Social Health Activist ) for promoting institutional delivery among
pregnant women . Under this initiative , eligible pregnant women are entitled to get
JSY benefit directly into their bank accounts.
 Government of India has launched the Janani Shishu Suraksha Karyakaram (JSSK )
on 1st June , 2011. The scheme is to benefit pregnant women who access
Government health facilities for their delivery . Moreover it will motivate those who
still choose to deliver at their homes to opt for institutional deliveries . All the States
and UTs have initiated implementation of the scheme .

81
SHANKAR TM

IAS ACADEMY
^ The Best IAS Academy In South India SINCE 2004

Key features of the scheme

 The initiative entitles all pregnant women delivering in public health institutions to
absolutely free and no expense delivery , including caesarean section .
 The entitlements include free drugs and consumables , free diet up to 3 days during
normal delivery and up to 7 days for C-section , free diagnostics, and free blood
wherever required . This initiative also provides for free transport from home to
institution , between facilities in case of a referral and drop back home . Similar
entitlements have been put in place for all sick newborns accessing public health
institutions for treatment till 30 days after birth .This has now been expanded to
cover sick infants:
 The scheme aims to eliminate out of pocket expenses incurred by the pregnant
women and sick new borne while accessing services at Government health facilities.
 The scheme is estimated to benefit more than 12 million pregnant women who
access Government health facilities for their delivery. Moreover it will motivate those
who still choose to deliver at their homes to opt for institutional deliveries.
 All the States and UTs have initiated implementation of the scheme
 Pradhan MantriMatruVandanaYojana ( PMMVY ) is a Maternity Benefit Programme
that is implemented in all the districts of the country in accordance with the
provision of the National Food Security Act , 2013.

Objectives

 Providing partial compensation for the wage loss in terms of cash incentive s so that
the woman can take adequate res t before and after delivery of the first living child .
 The cash incentive provided would lead to improved health seeking behaviour
amongst the Pregnant Women and Lactating Mothers ( PW & LM ) .
 POSHAN Abhiyaan is a multi-ministerial convergence mission with the vision to
ensure attainment of malnutrition free India by 2022 . The objective of POSHAN
Abhiyaan to reduce stunting in identified Districts of India with the highest
malnutrition burden by improving utilization of key Anganwadi Services and
improving the quality of Anganwadi Services delivery. Its aim to ensure holistic
development and adequate nutrition for pregnant women , mothers and children .

SOURCE:

https: / / www. thehindu . com / news / national / other-states / maternal-death- rate-declining-


report / article 29925365.ece

82
SHANKAR TM

IAS ACADEMY
4 The Best IAS Academy In South India SINCE 2004

83. Consider the following statements about the Jal Marg Vikas Project ( JMVP) :
1. It was aimed at the capacity augmentation of navigation on National Waterway -
1 ( NW - 1 ) .
2. It is being implemented by the Inland Waterways Authority of India under the
Ministry of Shipping with the support from the World Bank .
3. The states that are being covered under thisproject are Uttar Pradesh , Bihar ,
Jharkhand and West Bengal.
Which of the statements given above is / are correct?

(a) 1 only
( b) 1 and 2 only
(c) 2 and 3 only
(d ) 1 , 2 and 3

EXPLANATION:

Jal Marg Vikas Project (JMVP) is for capacity augmentation of navigation on National
Waterway- 1 ( NW- 1) at a cost of Rs 5369.18 crore with the technical assistance and
investment support of the World Bank . The Project is expected to be completed by
March , 2023. So , Statement 1 is correct.
For implementation of the Jal Marg Vikas Project , technical assistance and investment
support is being provided by the World Bank . Inland Waterways Authority of India (IWAI )
through selected Consultancy firms will conduct (i) Detailed Feasibility Study and
Detailed Engineering for Ancillary works on National Waterway-1; (ii) Environmental and
Social Impact Assessment ( ESIA) , Environmental Management Plan ( EMP) and
Resettlement Action Plan ( RAP) ; and (iii) IWT Sector Development Strategy and Market
Development Study during 2015- 16 . So , Statement 2 is correct.
States covered under the project are: Uttar Pradesh , Bihar , Jharkhand , West Bengal. So ,
statement 3 is correct.

ADDITIONAL INFORMATION:

Out of the 111 National Waterways ( NWs ) declared under the National Waterways Act ,
2016 , 13 NWs are operational for shipping and navigation and cargo / passenger vessels are
moving on them . The details of operational NWs are as follows:

83
SHANKAR TM

IAS ACADEMY
C The Best IAS Academy In South India SINCE 2004

Details of Operational National Waterways

SI. Length
National Waterway ( NAY) No. Location (S)
No. (km)

NAV-l: Ganga-Bhagirathi-Hooghly River System Uttar Pradesh, Bihar,


1. 1620
(Haldia - Allahabad) Jharkhand, West Bengal

Assam
2. NAY-2: Brahmaputra River (Dhubn - Sadiya) S91

NW-3: West Coast Canal (Kottapuram - Kollam),


3. 205
Champakara and Udyogmandal Canals

Kerala

NW-4: Phase- 1development of the stretch Muktiyala to


4. S2 Andhra Pradesh
Yijyawada of river Krishna

Waterways in Maharashtra

5.
i) NAV-10 (Amba River)
45
6. Maharashtra
ii) NAY-S3 (Rajpuri Creek )
31
7.
iii) NW-85 (Revadanda Creek - Kundalika River
31
System )

iv) NW-91 ( Shastri river-Jaigad creek system )


8.
52

National Waterways in Goa


9. 41
NW-68 - Mandovi - Usgaon Bridge to Arabian Sea
(41 km)

10. NW-111 —
Port (50 km).
Zuari- Sanvordem Bridge to Marmugao 50

Goa

84
SHANKAR TM

IAS ACADEMY
^ The Best IAS Academy In South India SINCE 2004

National Waterways in Gujarat

11. NW- 73- Narmada river- 226


Gujarat &Maharastra
12. NW-100- Tapi river 436

West Bengal
Sunderbans Waterways (NW-97): Namkhana to
13. 172
AtharaBankiKhal m West Bengal. (through Indo-Bangladesh
Protocol Route)

SOURCE:

https: / / pib . gov .in / newsite / PrintRelease .aspx?relid = 159372

84. Recently the Fourth Indian Ocean Conference 2019 was organized at Male , the
capital of Maldives . In this context , consider the following statements:
1. It was organised with the theme of ‘Securing the Indian Ocean Region:
Traditional and Non -Traditional Challenges’.
2. It was initiated by the Indian Ocean Rim Association ( IORA ) .
3. It will provide an opportunity for stakeholders of the Indian Ocean Region to
deliberate on building an institutional framework for managing the threats to
regional peace and human security .
Which of the statements given above are correct?

(a) 1 and 2 only


( b) 1 and 3 only
(c) 2 and 3 only
(d ) 1 , 2 and 3

EXPLANATION:

The 4 th edition of Indian Ocean Conference (IOC ) is being organised by India Foundation
in association with Government of Maldives and S . Rajaratnam School of International
Studies, Singapore on 03-04 September 2019 in Maldives. The theme of IOC 2019 is
"Securing the Indian Ocean Region: Traditional and Non -Traditional Challenges" . So ,
Statement 1 is correct .
Indian Ocean Conference is initiated by India Foundation along with its partners from

85
SHANKAR TM

IAS ACADEMY
^ The Best IAS Academy In South India SINCE 2004

Singapore , Sri Lanka and Bangladesh . So , Statement 2 is not correct .


The Conference provides opportunity for stakeholders of the Indian Ocean Region to
deliberate on building an institutional framework for managing the threats to regional
peace and human security through the strengthening of multilateral cooperation based
on mutual respect , equality and in line with international law. So , Statement 3 is
correct .

SOURCE:

http: / / indianocean .indiafoundation .in /

85 . Consider the following statements:


1 . In India , Union Government notifies minor minerals and State Governments
make Rules for grant of concessions in respect of extraction of minor minerals .
2. India has largest Coal reserves and does not import Coal .
3. Odisha has both Iron Ore and Manganese mines.
Which of the statements given above is / are correct?

(a) 1 and 2 only


( b) 2 only
(c ) 1 and 3 only
( d ) 3 only
EXPLANATION:

The central government has the power to notify “ minor minerals” under section 3 (e ) of
the MMDR Act , 1957 . On the other hand , as per Section 15 of the MMDR Act , 1957 State
Governments have complete powers for making Rules for grant of concessions in respect
of extraction of minor minerals and levy and collection of royalty on minor minerals . So ,
Statement 1 is correct .

India has the fourth largest coal reserves . India imports both non -coking and coking coal .
So , Statement 2 is not correct .

Odisha has both iron ore and manganese mines . So , Statement 3 is correct .

SOURCE:

https: / / economictimes .indiatimes.com / industry / indl- goods / svs / metals- mining / indias-
coal- import- rises-9 - pc- to- 234 - mt-in -fyl 9 / articleshow / 68990995. cms?from = mdr

86
SHANKAR TM

IAS ACADEMY
4 The Best IAS Academy In South India SINCE 2004

86. Which of the following river cross / crosses Standard Meridian of India ?
1. Mahanadi
2. Narmada
.
3 Son
.
4 Kosi
Select the correct answer using the code given below .
(a) 1 , 2 and 3 only
( b) 3 and 4 only
(c ) 1 and 2 only
( d ) 1 and 3 only

EXPLANATION:

87
FREE BOOKS, NOTES & VIDEOS FOR CIVILSERVICES

EBOOKS & UPSC PRELIMS USPC MAINS VIDEO FOR DAILY


MAGZINES MATERIALS MATERIALS CIVILSERVICES NEWSAPERS

SECUREIAS UPSC PRELIMS UPSC MAINS DELHI CIVILSERVICES


TESTSERIES TESTSERIES STUDENTS BOOKS

OPTIONAL SUBJECTS BOOKS, STATE PCS, SSC, BANKING


TEST SERIES, VIDEOS & NOTES BOOKS, TESTS VIDEOS & NOTES
1.GEOGRAPHY 1.UPPSC 2.SSC 3.MPSC
2.HISTORY 4.IBPS 5.RAS & RPSC
3.MATHEMATICS ENGINEERING BOOKS & MATERIAL
4. SOCIOLOGY 1. IES 2. GATE 3. IFoS
5.PUBLIC ADMINISTRATION 4. COMPUTER SCIENCE
6. POLITICAL SCIENCE 5. MECHINICAL ENGINEERING
7. ECONOMICS OTHER TELEGRAM CHANNELS
8 PHYSICS 1 GOVERNMENT JOBS
9 COMMERCE ACCOUNTANCY 2 LEARN YOGA & MEDITATION
10 ANTHROPOLOGY 3 LEARN ENGLISH
11 LAW 4 BEST DELAS & OFFERS
12 PHILOSOPHY 5 IAS HINDI BOOKS
13 CHARTERED ACCOUNTANTANCY 6 PDFs FOR ALL EXAMS
14 MEDICAL SCIENCE 7. WORLD DIGITAL LIBIRARY
1.CHENNAI STUDENTS 2.BANGLORE STUDENTS 3. CURRENT AFFAIRS
CONTACT FOR ADVERTISEMENT IN ABOVE CHANNLES
ADMIN1: ADMIN2:
SHANKAR TM

IAS ACADEMY
\ The Best IAS Academy In South India SINCE 2004

87. Consider the following left bank tributaries of river Ganga:


1 . Gandak
.
2 Kosi
.
3 Ghaghara
.
4 Gomti
Which among the following is the correct order of the above rivers from East to
West?
(a) 3- 4- 1- 2
(b ) 2-1-3-4
(c) 2-3- 1- 4
( d ) 1- 2 - 4- 3

EXPLANATION:

So , Option (b ) is correct.

88. Which of the following crop(s ) cultivated in Karewas, the lacustrine deposit
found in the union territory of Jammu and Kashmir?
.
1 Saffron
2. Rice
3. Maize
4. Walnut

Select the correct answer using the code given below .


88
SHANKAR TM

IAS ACADEMY
^ The Best IAS Academy In South India SINCE 2004

(a) 1 , 2 and 3 only


(b) 1 and 3 only
(c) 2 and 4 only
(d) 1 , 2 , 3 and 4
EXPLANATION:

Table U
-
Village Chandbara Landuse in Khan/ Season , 1994
Cultivated Land 961.87 acres
Net Cropped Land 900.62
Crop Area in acres Percentage to the Percentage to the
gross cropped area net cropped area
Saffron(pcrenniaJ crop) 659.38 68.55 -
73 21
Almond 59.00 6.13 6.55
Orchards 8.12 0.84 0.90
Almond and saffron
Rice
-
68 37 7.11 7.59
3927 4.15 4.43
Vegetables 28.50 2.96 3.16
Fodder 8.63 089 0.96
Mane 28.75 2.99 3.19
Fallow 61.25 6.37
Total 961.87 100.00 100.00

So , Option ( d ) is correct.

89. In which of the following region / regions Coal Bed Methane ( CBM ) can be
found?
1. Damodar Koel valley
2. Son valley
3. Jharia
4. KG basin

Select the correct answer using the code given below .

(a) 1 , 2 and 3 only


(b) 1 and 3 only
(c) 2 and 4 only
(d ) 1 , 2 , 3 and 4

89
SHANKAR TM

IAS ACADEMY
^ The Best IAS Academy In South India SINCE 2004

EXPLANATION:

All the current CBM producing blocks occur in the Gondwana sediments of eastern India
which host the bulk coal reserves of India . The majority of the best prospective areas for
CBM development are situated in Damodar Koel valley and Son valley .

CBM projects exist in south , east and north areas of Raniganj coalfield , the Parbatpur
block in Jharia coalfield and the East and west Bokaro coalfields , and north , east and west
blocks of Son valley. So , option ( d ) is correct.

90. Atmospheric conditions are well governed by humidity . Which among the
following best define ‘Humidity’?
(a) Form of suspended water droplets caused by condensation
( b ) Deposition of atmospheric moisture
( c ) Almost microscopically small droplets of water condensed from and suspended in air
( d ) The moisture content of the atmosphere at a particular time and place

EXPLANATION:

Humidity is the amount of water vapor in the air . If there is a lot of water vapor in the air ,
the humidity will be high . The higher the humidity , the wetter it feels outside .

On the weather reports, humidity is usually explained as relative humidity. Relative


humidity is the amount of water vapor actually in the air , expressed as a percentage of the
maximum amount of water vapor the air can hold at the same temperature . Think of the
air at a chilly - 10 degrees Celsius ( 14 degrees Fahrenheit) . At that temperature , the air can
hold , at most , 2.2 grams of water per cubic meter. So if there are 2.2 grams of water per
cubic meter when its - 10 degrees Celsius outside , were at an uncomfortable 100 percent
relative humidity. If there was 1.1 grams of water in the air at - 10 degrees Celsius, were at
50 percent relative humidity.

When humidity is high , the air is so clogged with water vapor that there isnt room for much
else . If you sweat when its humid , it can be hard to cool off because your sweat
cant evaporate into the air like it needs to.

SOURCE:

https: / / www.nationalgeographic.org / encyclopedia / humidity /

90
SHANKAR TM

IAS ACADEMY
^ The Best IAS Academy In South India SINCE 2004

91 . Which among the following statements is correct about proposed Samudrayaan


project?

(a) It is a project also known as SethuSamudram project


( b) It is a project by the Ministry of Shipping to connect all the major ports of the
country
(c) It is a mission of ISRO to send human into deep ocean bed of the Arctic ocean
(d ) It is a pilot project of the Ministry of Barth Sciences for deep ocean mining for
rare minerals
EXPLANATION:

The samudrayaan project proposes to send a submersible vehicle with three persons to a
depth of about 6000 metres to carry out deep underwater studies. The ' Samudrayaan ' is
a part of the Ministry of Earth Sciences ' pilot project for deep ocean mining for rare
minerals . So , option (c ) is correct .
Sethusamudram Shipping Canal Project is a proposed project to create a shipping route
in the shallow straits between India and Sri Lanka . This would provide a continuously
navigable sea route around the Indian Peninsula . So , option (a ) is not correct .
It is a pilot project of the Ministry of Earth Sciences . So , option ( b) is not correct .
The ' Samudrayaan ' project undertaken by the NIOT, Chennai and is not a mission of
ISRO . So , option (c ) is not correct.

ADDITIONAL INFORMATION:

If the ‘Samudrayaan ’ project is successful , India will join the league of developed nations in
the exploration of minerals from oceans. India could be the first developing country to
undertake such a project. The ' Samudrayaan ' project undertaken by the NIOT, Chennai , is
in line with the ISRO ' s ambitious ' Gaganyaan ' mission of sending an astronaut to space by
2022 . The ฀200 crore project proposes to carry three persons in a submersible vehicle to a
depth of 6000 metres under sea for various studies . Submarines go only about 200 metres .

SOURCE:

https: / / www.livemint.com /

92. Consider the following statements about the Montreal Protocol on Substances
that Deplete the Ozone Layer:
1. It is a protocol to the Vienna Convention for the Protection of the Ozone Layer.
2. The Protocol is to date the only UN treaty ever that has been ratified by all 197
UN Member States .

91
SHANKAR TM

IAS ACADEMY
^ The Best IAS Academy In South India SINCE 2004

3. Under the Kigali Amendment to this protocol , countries promised to reduce


the use of hydrofluorocarbons ( HFCs ) by more than 80% over the next 30
years .

Which of the statements given above are correct?

(a) 1 and 2 only


( b) 1 and 3 only
(c) 2 and 3 only
(d ) 1 , 2 and 3

EXPLANATION:

The Montreal Protocol on Substances that Deplete the Ozone Layer which is a protocol
to the Vienna Convention for the Protection of the Ozone Layer , is an international
treaty designed to protect the ozone layer by phasing out the production of numerous
substances that are responsible for ozone depletion . So , Statement 1 is correct .
The treaty has been ratified by 197 parties ( 196 states and the European Union ) ,
making it one of the first universally ratified treaties in United Nations history. So ,
Statement 2 is correct .
On October 15 , 2016 , Parties to the Montreal Protocol adopted the Kigali amendment to
phase down production and consumption of hydrofluorocarbons ( HFCs ) worldwide .
HFCs are widely used alternatives to ozone depleting substances such as
hydrochlorofluorocarbons ( HCFCs) and chlorofluorocarbons ( CFCs) , already controlled
under the Protocol. So, Statement 3 is correct .

ADDITIONAL INFORMATION:

 The Montreal Protocol on Substances that Deplete the Ozone Layer is the landmark
multilateral environmental agreement that regulates the production and
consumption of nearly 100 man- made chemicals referred to as ozone depleting
substances ( ODS ) . When released to the atmosphere , those chemicals damage the
stratospheric ozone layer , Earth ’s protective shield that protects humans and the
environment from harmful levels of ultraviolet radiation from the sun . Adopted on 15
September 1987 , the Protocol is to date the only UN treaty ever that has been
ratified every country on Earth - all 197 UN Member States.
 The Montreal Protocol phases down the consumption and production of the different
ODS in a step-wise manner , with different timetables for developed and developing
countries ( referred to as “Article 5 countries”) . Under this treaty , all parties have
specific responsibilities related to the phase out of the different groups of ODS ,
control of ODS trade , annual reporting of data, national licensing systems to control

92
SHANKAR TM

IAS ACADEMY
^ The Best IAS Academy In South India SINCE 2004

ODS imports and exports , and other matters. Developing and developed countries
have equal but differentiated responsibilities , but most importantly, both groups of
countries have binding , time-targeted and measurable commitments .
 The Multilateral Fund for the Implementation of the Montreal Protocol was
established in 1991 under Article 10 of the treaty . The Fund ' s objective is to provide
financial and technical assistance to developing country parties to the Montreal
Protocol whose annual per capita consumption and production of ODS is less than
0.3 kg to comply with the control measures of the Protocol.
 The Multilateral Fund ’s activities are implemented by four international agencies -
UN Environment ( UNEP) , UN Development Programme ( UNDP) , UN Industrial
Development Organisation ( UNIDO ) and the World Bank - as well as bilateral
agencies of non -Article 5 countries.

SOURCE:

https: / / www.state . gov / key- topics-office-of-environmental- quality-and-transboundary-


issues / the- montreal-protocol-on -substances-that- deplete-the-ozone-layer /

93. Consider the following features:


1. It is the oldest national park in India .
2. It was established to protect the endangered Bengal tiger.
3. It has sub -Himalayan belt geographical and ecological characteristics .

Which of the following protected areas has theabove given features?

(a ) Buxa Tiger Reserve


( b) Dachigam National Park
(c) Corbett National Park
(d ) Nanda Devi National Park

EXPLANATION:

 Jim Corbett National Park , which is a part of the larger Corbett Tiger Reserve , a
Project Tiger Reserve lies in the Nainital district of Uttarakhand . The magical
landscape of Corbett is well known and fabled for its tiger richness .
 Established in the year 1936 as Hailey National Park , Corbett has the glory of
being India ' s oldest and most prestigious National Park . It is also being honored
as the place where Project Tiger was first launched in 1973.
 This unique tiger territory is best known as the father who gave birth of the
Project Tiger in India to protect the most endangered species and the Royal of
India called Tigers.The park has sub- Himalayan belt geographical and ecological
characteristics .
So, option ( c ) is correct .

93
SHANKAR TM

IAS ACADEMY
^ The Best IAS Academy In South India SINCE 2004

SOURCE:

https: / / uttarakhandtourism .gov.in / activity / corbett- national- park- 2 /

94. The Mission Reach Out , sometimes seen in news , is related with
(a) Evacuation operation in the flood affected regions of Maharashtra
( b) Ensuring the basic necessities in the Jammu region
(c) Countering the terror attacks in reply to Pulwama attack
(d ) Humanitarian Assistance to Rohingya Muslims

EXPLANATION:

The Army had launched " Mission Reach Out " in Jammu to ensure basic necessities and
essential services are available in the region post the abrogation of the provisions of Article
370 and reorganisation of Jammu and Kashmir. So , Option (b) is correct.

SOURCE:

https: / / www.livemint . com / news / india / army-launches- mission - reach-out-in - jammu -


1565534164432 . html

95 . Consider the following pairs:

Straits Separates/Connects

1. Duncan North and South Andaman

2. Bab el Red sea and Gulf of Aden


Mandab
3. Taiwan South China sea and East China
sea

Which of the pairs given above are correctly matched?

(a) 1 and 4 only


(b ) 2 and 3 only
(c ) 1 and 3 only
( d ) 1, 2 and 3

94
SHANKAR TM

IAS ACADEMY
The Best IAS Academy In South India SINCE 2004

EXPLANATION:

Port Blair

N. Sentinel I

Tarmugh I
I
South
\ * Andaman
Labyritnth Is

Rutland I aCPhers
°n
Portman Bay
Twins

y Cinque Is

Passage I
^ Sisters I

Duncan Passage
South Sentinel I
North Brother I

South Brother I

Little
Andaman
Map of Yemen and the Bab el Mandeb Strait

95
SHANKAR TM

IAS ACADEMY
\ The Best IAS Academy In South India SINCE 2004

96 . The KajinSara lake , considered to be the World’s highest lake , is located at


(a) Bolivia
( b) India
(c) Nepal
(d ) Switcherland

EXPLANATION:

A newly-discovered lake in Nepal is likely to set a new record of being the world ' s
highest lake replacing Tilicho , which is situated at an altitude of 4 , 919 metres in the
Himalayan nation and currently holding the title . The Kajin Sara lake in Manang
district was discovered about a few months ago by a team of mountaineers . So ,
Option ( c ) is correct .

SOURCE:

https: / / www.ndtv.com / world - news / kajin -sara- newly- discovered- nepal-lake-likely- to-
become- worlds- highest- 2083265

97. Consider the following statements:


1 . Thorium reserves of India make upto 25 per cent of the global Thorium
reserves .
2. India imports Uranium from Russia , Kazakhstan , France , Canada and Australia .
3. State of Andhra Pradesh is the largest producer of Uranium in India .

96
SHANKAR TM

IAS ACADEMY
4 The Best IAS Academy In South India SINCE 2004

Select the correct answer using the code given below .

(a) 1 only
( b) 2 and 3 only
(c ) 1 and 3 only
( d ) 1 , 2 and 3
EXPLANATION:

India ' s thorium deposits , estimated at 360 , 000 tonnes, far outweigh its natural uranium
deposits at 70 ,000 tonnes . The country ' s thorium reserves make up 25 per cent of the
global reserves. So , Statement 1 is correct .

India currently imports uranium from Kazakhstan and Canada and plans to also
purchase the fuel from Australia . So , Statement 2 is not correct .

The state of Andhra Pradesh is the largest producer of uranium in India. Tummalapalle
village located in the Kadapa district of Andhra Pradesh is considered as one of
the largest uranium reserves in the world . So , Statement 3 is correct .

98 . Consider the following pairs:


Glacier Significance

1 . Siachen Longest glacier in Indian Himalayas .

2. Milam Source of River Mandakini

3. Zemu Sources of River Teesta

4. Sumeru Source of River Gori Ganga

Which of the pairs given above is / are correctly matched?

(a) 1 only

(b ) 1 and 3 only
(c ) 2 and 4 only
( d ) 1 , 3 and 4 only
EXPLANATION:

The Siachen in Jammu and Kashmir ( 18 , 000ft.) near the Karakoram Pass is the longest
glacier in the Indian Himalayas . It is 78km long and the source for the Nubra River that
97
SHANKAR TM

IAS ACADEMY
^ The Best IAS Academy In South India SINCE 2004

waters the Nubra Valley near Leh in the Trans Himalayas . So , Pair 1 is correctly
matched .
Milam and Gangotri are the major glaciers in the state of Uttarakhand . Milam is situated
in the Kumaon Himalayas , originating from the eastern slopes of the Trishuli . The river
Gori Ganga has its source here . So , Pair 2 is not correctly matched .
The Zemu glacier is the largest in the Eastern Himalayas in Sikkim . It is at the base of
the Kanchendzonga and is one of the sources for the Teesta that joins the Brahmaputra .
So , Pair 3 is correctly matched.
The River Mandakini has its source in the Sumeru glacier above the Kedarnath
Temple .So , Pair 4 is correctly matched .

SOURCE:

https: / / www.thehindu . com / society / history- and- culture / the - journey- of- the- himalayan -
glaciers / article 24339175.ece

99 . Consider the following statements:


1 . To cultivate , Tea requires a average temperature ranging from 21 ° C to 29 ° C,
the region should be receiving rainfall between 150-200 cm andthe soil should
be well drained .
2. In India , Tea is grown primarily in Assam , West Bengal , Tamil Nadu and Kerala .
3. The export of tea from India is greater than import in the last five years .
Which of the statements given above are correct?

(a) 1 and 2 only


( b) 2 and 3 only
(c ) 1 and 3 only

( d ) 1 , 2 and 3
EXPLANATION:

Temperature: 21° C to 29 ° C is ideal for the production of tea .


Rainfall: 150 to 250 cm of rainfallis required for tea .
The soil should be well drained and rich in humus . So ,Statement 1 is correct .
Some of the most exquisite and globally acclaimed varieties of Indian tea hail from
plantations in Darjeeling , Assam and Nilgiris, although several varieties of Indian tea are
also grown across other Indian states as well. So ,Statement 2 is correct .

98
SHANKAR TM

IAS ACADEMY
^ The Best IAS Academy In South India SINCE 2004

India is one of the world’s largest consumers of tea , with about three-fourths of the
country’s total produce consumed locally . India stands fourth in terms of tea exports after
Kenya (including neighbouring African countries) , China and Sri Lanka respectively . As of
2016 , India was the second largest producer in the world with production of 1 , 267 million
kgs . Major importers of Indian tea were Russia, Iran , USA , UK and UAE . India also imports
tea majorly from Nepal . But , the export of Indian tea is greater than its imports . So ,
Statement 3 is correct .

SOURCE:

https: / / www.teacoffeespiceoflndia.com / tea

https: / / www.ibef . org / exports / in dian - tea-industry aspx ,

100 . Consider the following characteristics of an area in India:


1 . Hot and humid climate .
2. Annual rainfall of 200 cm .
3. Hill slopes upto an altitude of 1100 metres .
4. Annual range of temperature is 150C to 300C.
Which among the following crops are most likely to found in the area with
characteristics described above?

(a) Virginia tobacco


( b) Cotton

(c ) Pepper
( d ) Mustard
EXPLANATION:

Black pepper is a plant of humid tropics requiring high rainfall and humidity. The hot and
humid climate of sub mountainous tracts of Western Ghats is ideal for its cultivation . It
grows successfully between 20 ° North and South latitude , and from sea level up to 1500 m
above sea level. The crop tolerates temperatures between 10 ° and 40 ° C . The ideal
temperature is 23 -32 ° C with an average of 28° C . Optimum soil temperature for root
growth is 26- 28° C . A well distributed annual rainfall of 125- 200 cm is considered ideal for
black pepper . Black pepper can be grown in a wide range of soils with a pH of 5.5 to 6.5,
though in its natural habitat it thrives well in red laterite soils . Virginia tobacco can not
exist above 125 cm rainfall . Cotton is grown in black soil and does not require 200cm
rainfall. Mustard is grown in relatively arid areas. So , option ( c ) is correct .

99
SHANKAR TM

IAS ACADEMY
^ The Best IAS Academy In South India SINCE 2004

ADDITIONAL INFORMATION:

Black pepper ( Piper nigrum L.) ( Family : Piperaceae ) is a perennial vine grown for its berries
extensively used as spice and in medicine . India is one of the major producer , consumer
and exporter of black pepper in the world . Black pepper is cultivated to a large extent in
Kerala, Karnataka and Tamil Nadu and to a limited extent in Maharashtra, North eastern
states and Andaman & Nicobar Islands. Kerala and Karnataka account for a major portion
of production of black pepper in the country.

SOURCE:

http: / / vikaspedia .in / agriculture / crop- production / package-of - practices / spices / black-
pepper

100

You might also like